You are on page 1of 41

G.R. No.

76217 September 14, 1989


GERMAN MANAGEMENT & SERVICES, INC., petitioner,
vs.
HON. COURT OF APPEALS and ERNESTO VILLEZA, respondents.

G.R. No. L-76216 September 14, 1989


GERMAN MANAGEMENT & SERVICES, INC., petitioner,
vs.
HON. COURT OF APPEALS and ORLANDO GERNALE, respondents.

FACTS:

 Spouses Cynthia and Manuel Roxas, are the owners of a parcel of land. The spouses executed a special power of attorney, authorizing petitioner
German Management Services to develop their property into a residential subdivision.

 Finding that part of the property was occupied by private respondents and twenty other persons, petitioner advised the occupants to vacate the
premises but the latter refused. However, petitioner proceeded with the development of the subject property which included the portions occupied
and cultivated by private respondents.

 Private respondents filed an action for forcible entry against petitioner before the Municipal Trial Court, alleging that they are mountainside farmers
and members of the Concerned Citizens of Farmer's Association and that they have occupied and tilled their farmholdings some twelve to fifteen
years prior to the promulgation of P.D. No. 27

 Petitioner also deprived private respondents of their property without due process of law by:
(1) forcibly removing and destroying the barbed wire fence enclosing their farmholdings without notice;
(2) bulldozing the rice, corn fruit bearing trees and other crops of private respondents by means of force, violence and intimidation, in violation of
P.D. 1038 and
(3) trespassing, coercing and threatening to harass, remove and eject private respondents from their respective farmholdings in violation of P.D.
Nos. 316, 583, 815, and 1028.

 MTC: dismissed the complaint for forcible entry of the private respondents

 RTC: sustained the dismissal by the dismissal by the MTC.

 CA: reversed the decisions of MTC and RTC and held that since private respondents were in actual possession of the property at the time they
were forcibly ejected by petitioner, private respondents have a right to commence an action for forcible entry regardless of the legality or illegality of
possession.

ISSUE: W/N Private respondents are entitled to file a forcible entry case against petitioner.

HELD: YES, Notwithstanding petitioner's claim that it was duly authorized by the owners to develop the subject property, private respondents, as
actual possessors, can commence a forcible entry case against petitioner because ownership is not in issue. Forcible entry is merely a quieting
process and never determines the actual title to an estate. Title is not involved.

It must be stated that regardless of the actual condition of the title to the property, the party in peaceable quiet possession shall not be
turned out by a strong hand, violence or terror. Thus, a party who can prove prior possession can recover such possession even against the owner
himself. Whatever may be the character of his prior possession, if he has in his favor priority in time, he has the security that entitles him to remain
on the property until he is lawfully ejected by a person having a better right by accion publiciana or accion reivindicatoria.

[G.R. No. 142668. August 31, 2004]


UNITED COCONUT PLANTERS BANK and LUIS MA. ONGSIAPCO, petitioners,
vs.
RUBEN E. BASCO, respondent.

FACTS:

1
 Respondent Basco had been employed with the petitioner United Coconut Planters Bank (UCPB) for seventeen (17) years. Respondent is a
stockholder of the said bank and also worked as an underwriter at the United Coconut Planters Life Association (Coco Life), a subsidiary of UCPB
and also solicited insurance policies from UCPB employees.

 However, the respondent received a letter from the UCPB informing him of the termination of his employment with the bank for grave abuse of
discretion and authority, and breach of trust in the conduct of his job as Bank Operations Manager of its Olongapo Branch.

 The respondent thereafter filed a complaint for illegal dismissal, non-payment of salaries, and damages against the bank in the National Labor
Relations Commission (NLRC), but he still frequented the UCPB main office in Makati City to solicit insurance policies from the employees
thereat. He also discussed the complaint he filed against the bank with the said employees.

 Petitioner’s adduced evidence, that Ongsiapco seen Basco talking to some of the trainees, wherein the bank also kept important records in that
floor. Ongsiapco was surprised because non-participants in the training were not supposed to be in the premises. Besides, the respondent had
been dismissed and had filed complaints against the bank with the NLRC. Ongsiapco was worried that bank records could be purloined and
employees could be hurt.

 The next day, Ongsiapco issued a Memorandum to Belanio, the Vice-President for Security Services, directing the latter not to allow the
respondent access to the bank premises near the working area. The said Memorandum was circulated by the Chief of Security to the security
guards and bank employees.

 Respondent requested to UCPB to allow him to enter into the bank premises but it was denied by Ongsiapco.

 But the respondent was undaunted, went to the UCPB Makati Branch to receive a check from Rene Jolo, a bank employee, and to deposit money
with the bank for a friend. He seated himself on a sofa fronting the teller’s booth where other people were also seated. Meanwhile, two security
guards approached the respondent.

 The guards showed him the Ongsiapco’s Memorandum and told him to leave the bank premises. The respondent pleaded that he be allowed to
finish his transaction before leaving. One of the security guards contacted the management and was told to allow the respondent to finish his
transaction with the bank.

 Jose Regino Casil, an employee of the bank who was in the 7th floor of the building, was asked by Rene Jolo to bring a check to the respondent,
who was waiting in the lobby in front of the tellers booth. Casil while at the elevator motioned the respondent to come and get the check, but the
security guard tapped the respondent on the shoulder and prevented the latter from approaching Casil. The latter then walked towards the
respondent and handed him the check from Jolo.

 Therefore, the respondent filed a complaint for damages against the petitioners UCPB and Ongsiapco in the RTC for barring him to the premises of
the bank.

 Trial Court: rendered judgment in favor of the respondent with the awards for moral and exemplary damages.

 CA: affirming the decision of the RTC with modifications. The CA deleted the awards for moral and exemplary damages, but ordered the petitioner
bank to pay nominal damages on its finding that latter abused its right when its security guards stopped the respondent from proceeding to the
working area near the ATM section to get the check from Casil.

ISSUE: W/N has the right to prohibit the respondent from all access to all bank premises.

HELD: YES, the bank has the right to exclude certain individuals from its premises or to limit their access thereto as to time, to protect, not only its
premises and records, but also the persons of its personnel and its customers/clients while in the premises.

Under Article 429 of the New Civil Code, which provides that: The owner or lawful possessor of a thing has the right to exclude any person from the
enjoyment and disposal thereof. For this purpose, he may use such force as may be reasonably necessary to repel or prevent an actual or
threatened unlawful physical invasion or usurpation of his property.

The petitioners contend that the provision which enunciates the principle of self-help applies when there is a legitimate necessity to personally or
through another, prevent not only an unlawful, actual, but also a threatened unlawful aggression or usurpation of its properties and records, and its personnel
and customers/clients who are in its premises. The petitioners assert that petitioner Ongsiapco issued his Memorandum dated November 15, 1995 because
the respondent had been dismissed from his employment for varied grave offenses; hence, his presence in the premises of the bank posed a
threat to the integrity of its records and to the persons of its personnel. Besides, the petitioners contend, the respondent, while in the bank
premises, conversed with bank employees about his complaint for illegal dismissal against the petitioner bank then pending before the Labor
Arbiter, including negotiations with the petitioner banks counsels for an amicable settlement of the said case.

2
After all, by its very nature, the business of the petitioner bank is so impressed with public trust; banks are mandated to exercise a higher degree of
diligence in the handling of its affairs than that expected of an ordinary business enterprise. Banks handle transactions involving millions of pesos and
properties worth considerable sums of money. The banking business will thrive only as long as it maintains the trust and confidence of its customers/clients.
Indeed, the very nature of their work, the degree of responsibility, care and trustworthiness expected of officials and employees of the bank is far greater than
those of ordinary officers and employees in the other business firms.

Hence, no effort must be spared by banks and their officers and employees to ensure and preserve the trust and confidence of the general public
and its customers/clients, as well as the integrity of its records and the safety and well-being of its customers/clients while in its premises. For the said
purpose, banks may impose reasonable conditions or limitations to access by non-employees to its premises and records, such as the exclusion
of non-employees from the working areas for employees, even absent any imminent or actual unlawful aggression on or an invasion of its
properties or usurpation thereof, provided that such limitations are not contrary to the law.

G.R. No. 168061, October 12, 2009


BANK OF THE PHILIPPINE ISLANDS, Petitioner.
vs.
TEOFILO P. ICOT, ANOLITA ICOT PILAPIL, LENNIE P. ICOT, VILMA ICOT CUYOS, RESTITUTO C. ICOT, FLORIDO A. CUYOS, CAYETANO GARBO,
TEODULA P. ICOT, YOLA P. ICOT, and HEIRS OF GENARO ICOT, namely: AMANCIO P. ICOT, HERMELINA ICOT, EVELYN ICOT GARBO, CARLOS
P. ICOT, RENATO P. ICOT, JOSEPHINE A. ICOT, AMELIA I. GARBO, and ROMMEL ICOT, Respondents.

FACTS:

 Spouses Vicente and Trinidad Velasco (spouses Velasco) obtained from petitioner Bank of the Philippine Islands (petitioner) a loan amounting to
P50,000, secured by a real estate mortgage over a parcel of land.

 The spouses Velasco failed to pay the loan, resulting in petitioner foreclosing the mortgaged property. During the auction sale, petitioner was the
highest bidder. The spouses Velasco failed to redeem the property during the one-year redemption period; hence, petitioner’s ownership was
consolidated, and a Definite Deed of Sale was issued in its favor.

 Teofilo Icot (respondent) and the late Genaro and Felimon Icot (predecessors-in-interest of the other respondents) claimed to have been in quiet,
open and continuous possession of the subject real property which they allegedly acquired from their father, Roberto Icot, through an extrajudicial
settlement of estate in 1964. Upon learning of the mortgage of the subject real property, respondents filed separate cases for quieting of title
against Velasco.

 RTC rendered Judgment based on a Compromise Agreement entered into by the parties.

 Petitioner and Velasco entered into a Contract to Sell wherein the BPI agreed to sell to the latter the subject real property payable within a year on
installment basis.

 Velasco failed to pay the amount due, prompting petitioner to cancel the Contract to Sell. In a letter, petitioner reiterated its cancellation of the
contract and requested Velasco to peacefully surrender possession of the subject property.

 Respondents Amancio P. Icot and Florido A. Cuyos wrote petitioner a letter offering to purchase the subject property, but the same was rejected by
petitioner for being too low.

 Petitioner filed with the RTC a Petition for the Issuance of a Writ of Possession.

 RTC granted the issuance of writ of possession to the petitioner and further, denied the Motion for reconsideration of the respondents.

 Respondent filed with the CA, rendered judgment granting the petition and reversing the decision of RTC.

ISSUE: W/N the petitioner as the purchaser is entitled to the issuance of a writ of possession of the subject property.

HELD: NO, The obligation of a court to issue an ex parte writ of possession in favor of the purchaser in an extrajudicial foreclosure sale ceases to be
ministerial once it appears that there is a third party in possession of the property who is claiming a right adverse to that of the debtor/mortgagor. We further
held, thus:

3
Under [Article 433 of the Civil Code], one who claims to be the owner of a property possessed by another must bring the appropriate
judicial action for its physical recovery.

The term judicial process could mean no less than an ejectment suit or reivindicatory action in which ownership claims of the contending
parties may be properly heard and adjudicated.

An ex parte petition for issuance of a possessory writ under Section 7 of Act 3135[, as amended,] is not, strictly speaking, a judicial process
as contemplated above. Even if the same may be considered a judicial proceeding for the enforcement of one's right of possession as purchaser in a
foreclosure sale, it is not an ordinary suit filed in court by which one party sues another for the enforcement or protection of a right, or the prevention or redress
of a wrong.

It should be emphasized that an ex parte petition for issuance of a writ of possession is a non-litigious proceeding authorized in an extrajudicial
foreclosure of mortgage pursuant to Act 3135, as amended. Unlike a judicial foreclosure of real estate mortgage under Rule 68 of the Rules of Court, any
property brought within the ambit of the act is foreclosed by the filing of a petition, not with any court of justice, but with the office of the sheriff of the province
where the sale is to be made.

As such, a third person in possession of an extrajudicially foreclosed realty, who claims a right superior to that of the original mortgagor, will have
no opportunity to be heard on his claim in a proceeding of this nature. It stands to reason, therefore, that such third person may not be dispossessed on the
strength of a mere ex parte possessory writ, since to do so would be tantamount to his summary ejectment, in violation of the basic tenets of due process.

Besides, as earlier stressed, Article 433 of the Civil Code, cited above, requires nothing less that an action for ejectment to be brought
even by the true owner. After all, the actual possessor of a property enjoys a legal presumption of just title in his favor, which must be overcome
by the party claiming otherwise.

Thus, the right of possession by a purchaser in an extrajudicial foreclosure of real property is recognized only as against the judgment debtor and
his successor-in-interest, but not as against persons whose right of possession is adverse to the latter. In this case, respondents are third parties in
possession of the subject real property, holding the same under a title adverse to that of the mortgagor/judgment obligor, Velasco. Respondents are claiming
title by virtue of an extrajudicial settlement of their father’s estate executed in 1964. Upon learning of the mortgage of the real property by Velasco to petitioner,
respondents filed a case for quieting of title against Velasco. The latter later acknowledged or recognized respondent’s ownership of the real property in the
Compromise Agreement executed by the parties in the quieting of title case. Velasco even agreed to undertake restitution of the subject property by
contracting anew with and repurchasing the foreclosed property from petitioner.

Petitioner’s right to issuance of a writ of possession cannot be invoked against respondents. Respondent’s possession of the subject real property
is legally presumed to be pursuant to a just title which petitioner may endeavor to overcome in a judicial proceeding for recovery of property.

G.R. No. 165952, July 28, 2008


ANECO REALTY AND DEVELOPMENT CORPORATION, Petitioner.
vs.
LANDEX DEVELOPMENT CORPORATION, Respondent.

FACTS:

 Fernandez Hermanos Development, Inc. (FHDI) is the original owner of a tract of land in San Francisco Del Monte, Quezon City. FHDI subdivided
the land into thirty-nine (39) lots. It later sold twenty-two (22) lots to petitioner Aneco and the remaining seventeen (17) lots to respondent Landex.

 Dispute arose when Landex started the construction of a concrete wall on one of its lots. To restrain construction of the wall, Aneco filed a
complaint for injunction with the RTC.

 Aneco later filed two (2) supplemental complaints seeking to demolish the newly-built wall and to hold Landex liable for two million pesos in
damages.

 Landex filed its Answer alleging, among others, that Aneco was not deprived access to its lots due to the construction of the concrete wall. Landex
claimed that Aneco has its own entrance to its property along Miller Street, Resthaven Street, and San Francisco del Monte Street. The Resthaven
access, however, was rendered inaccessible when Aneco constructed a building on said street.

4
 Landex also claimed that FHDI sold ordinary lots, not subdivision lots, to Aneco based on the express stipulation in the deed of sale that FHDI was
not interested in pursuing its own subdivision project.

 RTC rendered in favor of ANECO to stop the completion of the concrete wall and excavation of the road lot in question and if the same is already
completed, to remove the same and to return the lot to its original situation.

 Acting on the motion of Landex, the RTC set a hearing on the motion for reconsideration on August 28, 1996. Aneco failed to attend the slated
hearing. The RTC gave Aneco additional time to file a comment on the motion for reconsideration.

 RTC issued an order denying the motion for execution of Aneco.

 RTC issued an order granting the motion for reconsideration of Landex and dismissing the complaint of Aneco. In granting reconsideration, the
RTC stated: that plaintiff could easily make an access to a public road within the bounds and limits of its own property.

 Aneco appealed to the CA, however, it was denied by the CA affirming the decision of the RTC and held that Aneco knew at the time of the sale
that the lots sold by FHDI were not subdivision units based on the express stipulation in the deed of sale that FHDI, the seller, was no longer
interested in pursuing its subdivision project.

ISSUE: W/N Aneco may prohibit Landex from constructing a concrete wall on its own property.

HELD: NO, the court agree with the RTC and the CA that the complaint for injunction against Landex should be dismissed for lack of merit. What is
involved here is an undue interference on the property rights of a landowner to build a concrete wall on his own property. It is a simple case of a
neighbor, petitioner Aneco, seeking to restrain a landowner, respondent Landex, from fencing his own land.

Article 430 of the Civil Code gives every owner the right to enclose or fence his land or tenement by means of walls, ditches, hedges or
any other means. The right to fence flows from the right of ownership. As owner of the land, Landex may fence his property subject only to the
limitations and restrictions provided by law. Absent a clear legal and enforceable right, as here, We will not interfere with the exercise of an
essential attribute of ownership.

Aneco cannot rely on the road lot under the old subdivision project of FHDI because it knew at the time of the sale that it was buying ordinary lots,
not subdivision lots, from FHDI. This is clear from the deed of sale between FHDI and Aneco where FHDI manifested that it was no longer interested in
pursuing its own subdivision project. If Aneco wants to transform its own lots into a subdivision project, it must make its own provision for road lots. It certainly
cannot piggy back on the road lot of the defunct subdivision project of FHDI to the detriment of the new owner Landex. The RTC and the CA correctly
dismissed the complaint for injunction of Aneco for lack of merit.

G.R. No. 116100. February 9, 1996


SPOUSES CRISTINO and BRIGIDA CUSTODIO and SPOUSES LITO and MARIA CRISTINA SANTOS, petitioners,
vs.
COURT OF APPEALS, HEIRS OF PACIFICO C. MABASA and REGIONAL TRIAL COURT OF PASIG, METRO MANILA, BRANCH 181, respondents.

FACTS:
 The original plaintiff Pacifico Mabasa died during the pendency of this case and was substituted by Ofelia Mabasa, his surviving spouse [and
children].

 The plaintiff owns a parcel of land with a two-door apartment erected thereon. The plaintiff was able to acquire said property through a contract of
sale with spouses Mamerto Rayos and Teodora Quintero as vendors. Said property may be described to be surrounded by other immovables
pertaining to defendants herein.

 As an access to P. Burgos Street from plaintiffs property, there are two possible passageways. The first passageway is approximately one meter
wide and is about 20 meters distant from Mabasa’s residence to P. Burgos Street. Such path is passing in between the row of houses. The second
passageway is about 3 meters in width and length from plaintiff Mabasa’s residence to P. Burgos Street; it is about 26 meters. In passing thru said
passageway, a less than a meter wide path through the septic tank and with 5-6 meters in length has to be traversed.

 When said property was purchased by Mabasa, there were tenants occupying the premises and who were acknowledged by plaintiff Mabasa as
tenants. However, one of said tenants vacated the apartment and when plaintiff Mabasa went to see the premises, he saw that there had been built
an adobe fence in the first passageway making it narrower in width. Said adobe fence was first constructed by defendants Santoses along their
property which is also along the first passageway.

5
 Defendant Morato constructed her adobe fence and even extended said fence in such a way that the entire passageway was enclosed. And it was
then that the remaining tenants of said apartment vacated the area.

 Defendant Ma. Cristina Santos testified that she constructed said fence because there was an incident when her daughter was dragged by a
bicycle pedalled by a son of one of the tenants in said apartment along the first passageway. She also mentioned some other inconveniences of
having the front of her house a pathway such as when some of the tenants were drunk and would bang their doors and windows. Some of their foot
wears were even lost.

 TRIAL COURT: rendered a decision, ordering defendants Custodios and Santoses to give plaintiff permanent access - ingress and egress, to the
public street.

 CA: Affirmed the decision of the trial court with modification and also denied the motion for reconsideration of the petitioners.

ISSUE: W/N the constructing of adobe fences by the petitioners within their lots is valid.
HELD: YES, the act of petitioners in constructing a fence within their lot is a valid exercise of their right as owners, hence not contrary to morals, good
customs or public policy. The law recognizes in the owner the right to enjoy and dispose of a thing, without other limitations than those established by law.

It is within the right of petitioners, as owners, to enclose and fence their property. Article 430 of the Civil Code provides that every owner may
enclose or fence his land or tenements by means of walls, ditches, live or dead hedges, or by any other means without detriment to servitudes
constituted thereon.

At the time of the construction of the fence, the lot was not subject to any servitudes. There was no easement of way existing in favor of
private respondents, either by law or by contract. The fact that private respondents had no existing right over the said passageway is confirmed by the
very decision of the trial court granting a compulsory right of way in their favor after payment of just compensation. It was only that decision which gave private
respondents the right to use the said passageway after payment of the compensation and imposed a corresponding duty on petitioners not to interfere in the
exercise of said right.

Hence, prior to said decision, petitioners had an absolute right over their property and their act of fencing and enclosing the same was an
act which they may lawfully perform in the employment and exercise of said right. To repeat, whatever injury or damage may have been sustained
by private respondents by reason of the rightful use of the said land by petitioners is damnum absque injuria.

A person has a right to the natural use and enjoyment of his own property, according to his pleasure, for all the purposes to which such property is
usually applied. As a general rule, therefore, there is no cause of action for acts done by one person upon his own property in a lawful and proper manner,
although such acts incidentally cause damage or an unavoidable loss to another, as such damage or loss is damnum absque injuria. When the owner of
property makes use thereof in the general and ordinary manner in which the property is used, such as fencing or enclosing the same as in this case, nobody
can complain of having been injured, because the inconvenience arising from said use can be considered as a mere consequence of community life.

The proper exercise of a lawful right cannot constitute a legal wrong for which an action will lie, although the act may result in damage to another,
for no legal right has been invaded. One may use any lawful means to accomplish a lawful purpose and though the means adopted may cause damage to
another, no cause of action arises in the latters favor. Any injury or damage occasioned thereby is damnum absque injuria. The courts can give no redress for
hardship to an individual resulting from action reasonably calculated to achieve a lawful end by lawful means.

G.R. No. 4223 August 19, 1908


NICOLAS LUNOD, ET AL., plaintiffs-appellees,
vs.
HIGINO MENESES, defendant-appellant.

FACTS:
 Nicolas Lunod, Juan de la Vega, Evaristo Rodriguez, Fernando Marcelo, Esteban Villena, Benito Litao, Ventura Hernandez, and Casimiro
Pantanilla, residents of the town of Bulacan, filed a written complaint against Higino Meneses, alleging that they each owned and possessed farm
lands, situated in the places known as Maytunas and Balot, near a small lake named Calalaran.

 Meneses is the owner of a fish-pond and a strip of land situated in Paraanan, adjoining the said lake on one side, and the River Taliptip on the
other.

 For more than twenty years before 1901, there existed and still exists in favor of the rice fields of the plaintiffs a statutory easement permitting the
flow of water over the said land in Paraanan, which easement the said plaintiffs enjoyed until the year 1901 and consisted in that the water
collected upon their lands and in the Calalaran Lake flow through Paraanan into the Taliptip River.

6
 From that year however, the defendant, without any right or reason, converted the land in Paraanan into a fishpond and by means of a dam and a
bamboo net, prevented the free passage of the water through said place into the Taliptip River, that in consequence the lands of the plaintiff
became flooded and damaged by the stagnant waters, there being no outlet except through the land in Paraanan;

 Therefore, plantation of the plaintiffs were destroyed, causing the loss and damages to the extent of about P1,000, and will continue if the
obstructions to the flow of the water are allowed to remain, preventing its passage through said land and injuring the rice plantations of the plaintiffs.

 The plaintiffs asked that judgment be entered against the defendant, declaring that the said tract of land in Paraanan is subject to a statutory
easement permitting the flow of water from the property of the plaintiffs, and that the defendant be ordered to remove and destroy the obstructions
that impede the passage of the waters through Paraanan.

 Defendant filed an answer, denying each allegations of the complaint, and alleged that no statutory easement existed nor could exist in favor of the
lands described in the complaint, permitting the waters to flow over the fish pond that he and his brothers, owned the area and boundaries which
they inherited from their deceased mother.

ISSUE: W/N Menesses have the right to fence his estate which causes the damages and loss to the plaintiff’s plantation.

HELD: NO, Meneses, had no right to construct the works, nor the dam which blocks the passage, through his lands and the outlet to the Taliptip
River, of the waters which flood the higher lands of the plaintiffs; and having done so, to the detriment of the easement charged on his estate, he
has violated the law which protects and guarantees the respective rights and regulates the duties of the owners of the fields in Calalaran and
Paraanan.

It is true that article 388 of said code authorizes every owner to enclose his estate by means of walls, ditches fences or any other device,
but his right is limited by the easement imposed upon his estate.

The defendant Meneses might have constructed the works necessary to make and maintain a fish pond within his own land, but he was always under the
strict and necessary obligation to respect the statutory easement of waters charged upon his property, and had no right to close the passage and
outlet of the waters flowing from the lands of the plaintiffs and the lake of Calalaran into the Taliptip River. He could not lawfully injure the owners
of the dominant estates by obstructing the outlet to the Taliptip River of the waters flooding the upper lands belonging to the plaintiffs.

It is perhaps useful and advantageous to the plaintiffs and other owners of high lands in Calalaran, in addition to the old dike between the lake of
said place and the low lands in Paraanan, to have another made by the defendant at the border of Paraanan adjoining the said river, for the purpose of
preventing the salt waters of the Taliptip River flooding, at high tide, not only the lowlands in Paraanan but also the higher ones of Calalaran and its lake, since
the plaintiffs can not prevent the defendant from protecting his lands against the influx of salt water; but the defendant could never be permitted to obstruct the
flow of the waters through his lands to the Taliptip River during the heavy rains, when the high lands in Calalaran and the lake in said place are flooded,
thereby impairing the right of the owners of the dominant estates.

MARGARITA F. CASTRO vs. NAPOLEON A. MONSOD


GR. No. 183719 | February 2, 2011
FACTS:
 Petitioner Margarita Castro is the registered owner of a parcel of land located on Garnet Street, Manuela Homes, Pamplona, Las Pinas City
 Respondent Napoleon Monsod, on the other hand, is the owner of the property adjoining the lot of petitioner, located on Lyra
Street, Moonwalk Village, Phase 2, Las Pinas City.
 There is a concrete fence, more or less two (2) meters high, dividing Manuela Homes from Moonwalk Village.
 Respondent caused the annotation of an adverse claim against sixty-five (65) sq.m. of the property of petitioner. The adverse claim was filed
without any claim of ownership over the property. Respondent was merely asserting the existing legal easement of lateral and subjacent support at
the rear portion of his estate to prevent the property from collapsing, since his property is located at an elevated plateau of fifteen (15) feet, more or
less, above the level of petitioner’s property.
 In defiance, petitioner filed a complaint for damages with temporary restraining order/writ of preliminary injunction. She also prayed that the
Register of Deeds of Las Pinas City be ordered to cancel the annotation of the adverse claim.
 Prior to the filing of the case before the RTC, there were deposits of soil and rocks about two (2) meters away from the front door of the house of
petitioner. As such, petitioner was not able to park her vehicle at the dead-end portion of Garnet Street.
 When petitioner noticed a leak that caused the front portion of her house to be slippery, she hired construction workers to see where the leak was
coming from. The workers had already started digging when police officers sent by respondent came and stopped the workers from finishing their
job
 Petitioner averred that when she bought the property from Manuela Homes in 1994, there was no annotation or existence of any easement over the
property.
 For his part, respondent claimed that he and his family had been residing in Moonwalk Village since June 1984, that the land elevation
of Moonwalk Village was almost on the same level as Manuela Homes.

7
 The trial court ratiocinated that the adverse claim of respondent was non-registrable considering that the basis of his claim was an easement and
not an interest adverse to the registered owner, and neither did he contest the title of petitioner. Furthermore, the adverse claim of respondent
failed to comply with the requisites provided under Section 70 of Presidential Decree No. 1529.

ISSUE: Whether the easement of lateral and subjacent support exists on the subject adjacent properties and, if it does, whether the same may be annotated
at the back of the title of the servient estate.

RULING:
Article 437 of the Civil Code provides that the owner of a parcel of land is the owner of its surface and of everything under it, and he can construct
thereon any works, or make any plantations and excavations which he may deem proper.
However, such right of the owner is not absolute and is subject to the following limitations:
(1) servitudes or easements,
(2) special laws,]
(3) ordinances,
(4) reasonable requirements of aerial navigation, and
(5) rights of third persons.

Respondents assertion that he has an adverse claim over the 65 sq.m. property of petitioner is misplaced since he does not have a claim over the
ownership of the land. The annotation of an adverse claim over registered land under Section 70 of Presidential Decree 1529 requires a claim on the
title of the disputed land. Annotation is done to apprise third persons that there is a controversy over the ownership of the land and to preserve and protect
the right of the adverse claimant during the pendency of the controversy. It is a notice to third persons that any transaction regarding the disputed land is
subject to the outcome of the dispute.

In reality, what respondent is claiming is a judicial recognition of the existence of the easement of subjacent and lateral support over the 65 sq. m.
portion of petitioner’s property covering the land support/embankment area. His reason for the annotation is only to prevent petitioner from removing
the embankment or from digging on the property for fear of soil erosion that might weaken the foundation of the rear portion of his property which is
adjacent to the property of petitioner.

An easement or servitude is an encumbrance imposed upon an immovable for the benefit of another immovable belonging to a different
owner. There are two kinds of easements according to source. An easement is established either by law or by will of the owners. The courts cannot
impose or constitute any servitude where none existed. They can only declare its existence if in reality it exists by law or by the will of the owners.
There are therefore no judicial easements.

Article 684 of the Civil Code provides that no proprietor shall make such excavations upon his land as to deprive any adjacent land or building of sufficient
lateral or subjacent support. An owner, by virtue of his surface right, may make excavations on his land, but his right is subject to the limitation that he shall not
deprive any adjacent land or building of sufficient lateral or subjacent support. Between two adjacent landowners, each has an absolute property right to have
his land laterally supported by the soil of his neighbor, and if either, in excavating on his own premises, he so disturbs the lateral support of his neighbors land
as to cause it, or, in its natural state, by the pressure of its own weight, to fall away or slide from its position, the one so excavating is liable.

In the instant case, an easement of subjacent and lateral support exists in favor of respondent. It was established that the properties of petitioner and
respondent adjoin each other. The residential house and lot of respondent is located on an elevated plateau of fifteen (15) feet above the level of petitioners
property. The embankment and the riprapped stones have been in existence even before petitioner became the owner of the property. It was proven that
petitioner has been making excavations and diggings on the subject embankment and, unless restrained, the continued excavation of the embankment could
cause the foundation of the rear portion of the house of respondent to collapse, resulting in the destruction of a huge part of the family dwelling

REPUBLIC OF THE PHILIPPINES (DIRECTOR OF FOREST DEVELOPMENT), petitioner,


vs.
HON. COURT OF APPEALS (THIRD DIVISION) and JOSE Y. DE LA ROSA, respondents.

G.R. No. L-44081 April 15, 1988

FACTS:
 Jose dela Rosa filed an application for registration of a parcel of land in Itogon Benguet, on his own behalf and on behalf of his children --- Victoria,
Benjamin and Eduardo.
 This application was separately opposed by Benguet Consolidated, Inc. (Benguet) and Atok Big Wedge Corporation (Atok).
 The petitioners claimed that they have acquired the land from their parents and that they have been in possession of the land ever since.
 Benguet and Atok opposed on the ground that they have mineral claims covering the property and had been in actual, continuous and exclusive
possession of the land in concept of owner.
 The trial court denied the application while the Court of Appeals reversed the decision of the trial court and recognized the claims of the applicant
but subject to the rights of Benguet and Atok respecting their mining claims.
 In other words, the Court of Appeals affirmed the surface rights of the de la Rosas over the land while at the same time reserving the sub-surface
rights of Benguet and Atok by virtue of their mining claims.

8
ISSUE: Whether or not the CA is correct on affirming the surface rights of the de la Rosas

RULING:
The CA was incorrect.
Our holding is that Benguet and Atok have exclusive rights to the property in question by virtue of their respective mining claims which they validly acquired
before the Constitution of 1935 prohibited the alienation of all lands of the public domain except agricultural lands, subject to vested rights existing at the time
of its adoption. The land was not and could not have been transferred to the private respondents by virtue of acquisitive prescription, nor could its use be
shared simultaneously by them and the mining companies for agricultural and mineral purposes.

Art. 437. The owner of a parcel of land is the owner of its surface and of everything under it, and he can construct thereon any works or make any
plantations and excavations which he may deem proper, without detriment to servitudes and subject to special laws and ordinances. He cannot
complain of the reasonable requirements of aerial navigation.

Under the theory of the respondent court, the surface owner will be planting on the land while the mining locator will be boring tunnels underneath. The farmer
cannot dig a well because he may interfere with the operations below and the miner cannot blast a tunnel lest he destroy the crops above. How deep can the
farmer, and how high can the miner, go without encroaching on each other's rights? Where is the dividing line between the surface and the sub-surface rights?

It is a well-known principle that the owner of piece of land has rights not only to its surface but also to everything underneath and the airspace above it up to a
reasonable height.

The rights over the land are indivisible and that the land itself cannot be half agricultural and half mineral. The classification must be categorical; the land
must be either completely mineral or completely agricultural.

In the instant case, as already observed, the land which was originally classified as forest land ceased to be so and became mineral — and
completely mineral — once the mining claims were perfected. As long as mining operations were being undertaken thereon, or underneath, it did not
cease to be so and become agricultural, even if only partly so, because it was enclosed with a fence and was cultivated by those who were unlawfully
occupying the surface.
This is an application of the Regalian doctrine which, as its name implies, is intended for the benefit of the State, not of private persons. The rule simply
reserves to the State all minerals that may be found in public and even private land devoted to "agricultural, industrial, commercial, residential or (for) any
purpose other than mining." Thus, if a person is the owner of agricultural land in which minerals are discovered, his ownership of such land does not give him
the right to extract or utilize the said minerals without the permission of the State to which such minerals belong.

JOSE D. SANGALANG and LUTGARDA D. SANGALANG, petitioners, FELIX C. GASTON and DOLORES R. GASTON, JOSE V. BRIONES and ALICIA
R. BRIONES, and BEL-AIR VILLAGE ASSOCIATION, INC., intervenors-petitioners,
vs.
INTERMEDIATE APPELLATE COURT and AYALA CORPORATION, respondents.

G.R. No. 71169 August 30, 1989

FACTS:

 Bel-Air Village is located north of Buendia Avenue extension (now Sen. Gil J. Puyat Ave.)
 Bel-Air Village was owned and developed into a residential subdivision in the 1950s by Makati Development Corporation (hereinafter referred to as
MDC), which in 1968 was merged with appellant Ayala Corporation.
 Appellees-spouses Sangalang reside at No. 11O Jupiter Street between Makati Avenue and Reposo Street;
 while appellee Bel-Air Village Association, Inc. (hereinafter referred to as BAVA) is the homeowners' association in Bel-Air Village which takes care
of the sanitation, security, traffic regulations and general welfare of the village.
 The lots which were acquired by appellees Sangalang and spouse Gaston and spouse and Briones and spouse in 1960, 1957 and 1958,
respectively, were all sold by MDC subject to certain conditions and easements contained in Deed Restrictions which formed a part of each deed of
sale.
 When MDC sold the lots to appellees' predecessors-in-interest, the whole stretch of the commercial block between Buendia Avenue and Jupiter
Street, from Reposo Street in the west to Zodiac Street in the east, was still undeveloped.
 Access, therefore, to Bel-Air Village was opened to all kinds of people and even animals. So in 1966, although it was not part of the original plan,
MDC constructed a fence or wall on the commercial block along Jupiter Street.
 In 1970, the fence or wall was partly destroyed by typhoon "Yoling." The destroyed portions were subsequently rebuilt by the appellant.
 When Jupiter Street was widened in 1972 by 3.5 meters, the fence or wall had to be destroyed. Upon request of BAVA, the wall was rebuilt inside
the boundary of the commercial block.

 The petitioner originally brought the complaint in the Regional Trial Court of Makati, principally for specific performance, plaintiff [now, petitioner]
alleging that the defendant [now, private respondent] Tenorio allowed defendant [Tenorio's co-private respondent] Gonzalves to occupy and
convert the house at 50 Jupiter Street, Bel-Air Village, Makati, Metro Manila, into a restaurant, without its knowledge and consent, and in
violation of the deed restrictions which provide that the lot and building thereon must be used only for residential purposes upon which the

9
prayed for main relief was for 'the defendants to permanently refrain from using the premises as commercial and to comply with the terms of the
Deed Restrictions."
 The trial court adjudged in favor of BAVA. On appeal, the CA reversed.
In brief, G.R. Nos. 74376, 76394, 78182, and 82281 are efforts to enforce the "deed restrictions" in question against specific residents (private respondents in
the petitions) of Jupiter Street and with respect to G.R. No. 78182, Reposo Street. The private respondents are alleged to have converted their residences into
commercial establishments (a restaurant in G.R. No. 74376, a bakery and coffee shop in G.R. No. 76394, an advertising firm in G.R. No. 78182; and a
construction company, apparently, in G.R. No. 82281) in violation of the said restrictions. 24

Their mother case, G. R. No. 71169 is, on the other hand, a petition to hold the vendor itself, Ayala Corporation (formerly Makati Development Corporation),
liable for tearing down the perimeter wall along Jupiter Street that had therefore closed its commercial section from the residences of Bel-Air Village and
ushering in, as a consequence, the full "commercialization" of Jupiter Street, in violation of the very restrictions it had authored.

ISSUE: Whether or not Ayala Corp may be held liable for damages for the commitment in maintaining the wall

RULING:
We come to the perimeter wall then standing on the commercial side of Jupiter Street the destruction of which opened the street to the public. The petitioners
contend that the opening of the thoroughfare had opened, in turn, the floodgates to the commercialization of Bel-Air Village. The wall, so they allege, was
designed precisely to protect the peace and privacy of Bel-Air Village residents from the din and uproar of mercantile pursuits, and that the Ayala Corporation
had committed itself to maintain it. It was the opinion of the Court of Appeals, as we said, that Ayala's liability therefor, if one existed, had been overtaken by
the passage of Ordinances Nos. 81 and 82-01, opening Jupiter Street to commerce.

It is our ruling, we reiterate, that Jupiter Street lies as a mere boundary, a fact acknowledged by the authorities of Makati and the National
Government and, as a scrutiny of the records themselves reveals, by the petitioners themselves, as the articles of incorporation of Bel-Air Village
Association itself would confirm. As a consequence, Jupiter Street was intended for the use by both -the commercial and residential blocks. It was not
originally constructed, therefore, for the exclusive use of either block, least of all the residents of Bel-Air Village, but, we repeat, in favor of both, as
distinguished from the general public.

When the wall was erected in 1966 and rebuilt twice, in 1970 and 1972, it was not for the purpose of physically separating the two blocks. According to Ayala
Corporation, it was put up to enable the Bel-Air Village Association "better control of the security in the area, 41 and as the Ayala Corporation's "show of
goodwill " 42 a view we find acceptable in the premises. For it cannot be denied that at that time, the commercial area was vacant, "open for [sic] animals and
people to have access to Bel-Air Village." 43 There was hence a necessity for a wall.

In any case, we find the petitioners' theory, that maintaining the wall was a matter of a contractual obligation on the part of Ayala, to be pure
conjecture. The records do not establish the existence of such a purported commitment. For one, the subdivision plans submitted did not mention
anything about it. For another, there is nothing in the "deed restrictions" that would point to any covenant regarding the construction of a wall. There is no
representation or promise whatsoever therein to that effect.

With the construction of the commercial buildings in 1974, the reason for which the wall was built- to secure Bel-Air Village from interlopers had
naturally ceased to exist. The buildings themselves had provided formidable curtains of security for the residents. It should be noted that the commercial lot
buyers themselves were forced to demolish parts of the wall to gain access to Jupiter Street, which they had after all equal right to use.

In fine, we cannot hold the Ayala Corporation liable for damages for a commitment it did not make, much less for alleged resort to machinations in evading it.

LUCIANO BRIONES and NELLY BRIONES,

Petitioners, vs.
JOSE MACABAGDAL, FE D. MACABAGDAL and VERGON REALTY INVESTMENTS CORPORATION,
Respondents.
G.R. No. 150666 | August 3, 2010

FACTS:
 Respondent-spouses purchased from Vergon Realty Investments Corporation (Vergon) Lot No. 2-R, a 325-square-meter land located in
Vergonville Subdivision No. 10 at Las Pias City
 Sometime in 1984, after obtaining the necessary building permit and the approval of Vergon, petitioners constructed a house on Lot No. 2-R which
they thought was Lot No. 2-S.
 After being informed of the mix up by Vergons manager, respondent-spouses immediately demanded petitioners to demolish the house and vacate
the property.
 Petitioners, however, refused to heed their demand. Thus, respondent-spouses filed an action to recover ownership and possession of the said
parcel of land with the RTC of Makati City.
 Petitioners insisted that the lot on which they constructed their house was the lot which was consistently pointed to them as theirs by Vergons
agents over the seven (7)-year period they were paying for the lot.

10
 They interposed the defense of being buyers in good faith and impleaded Vergon as third-party defendant claiming that because of the warranty
against eviction, they were entitled to indemnity from Vergon in case the suit is decided against them.
 The RTC ruled in favor of respondent-spouses and found that petitioners house was undoubtedly built on Lot No. 2-R.
 On appeal, the CA affirmed the RTCs finding that the lot upon which petitioners built their house was not the one (1) which Vergon sold to them.
o Based on the documentary evidence, such as the titles of the two (2) lots, the contracts to sell, and the survey report made by the
geodetic engineer, petitioners house was built on the lot of the respondent-spouses. here was no basis to presume that the error was
Vergons fault.
o The CA further ruled that petitioners cannot use the defense of allegedly being a purchaser in good faith for wrongful occupation of land
 Petitioner’s contention
o that they must not bear the damage alone.
o that they relied with full faith and confidence in the reputation of Vergons agents when they pointed the wrong property to them. Even
the President of Vergon, Felix Gonzales, consented to the construction of the house when he signed the building permit.
o That they are builders in good faith.

ISSUE: Whether or not the petitioners are indeed builders in good faith

RULING:
We find that the trial court nonetheless erred in outrightly ordering petitioners to vacate the subject property or to pay respondent spouses the prevailing price
of the land as compensation. Article 527 of the Civil Code presumes good faith, and since no proof exists to show that the mistake was done by petitioners in
bad faith, the latter should be presumed to have built the house in good faith.

When a person builds in good faith on the land of another, Article 448 of the Civil Code governs. Said article provides,
ART. 448. The owner of the land on which anything has been built, sown or planted in good faith, shall have the right to appropriate as his own
the works, sowing or planting, after payment of the indemnity provided for in Articles 546 and 548, or to oblige the one who built or planted to
pay the price of the land, and the one who sowed, the proper rent. However, the builder or planter cannot be obliged to buy the land if its value is
considerably more than that of the building or trees. In such case, he shall pay reasonable rent, if the owner of the land does not choose to
appropriate the building or trees after proper indemnity. The parties shall agree upon the terms of the lease and in case of disagreement, the court
shall fix the terms thereof.

The above-cited article covers cases in which the builders, sowers or planters believe themselves to be owners of the land or, at least, to have a claim of title
thereto.

The builder in good faith can compel the landowner to make a choice between appropriating the building by paying the proper indemnity or
obliging the builder to pay the price of the land. The choice belongs to the owner of the land, a rule that accords with the principle of
accession, i.e., that the accessory follows the principal and not the other way around.

However, even as the option lies with the landowner, the grant to him, nevertheless, is preclusive. He must choose one. He cannot, for instance, compel
the owner of the building to remove the building from the land without first exercising either option. It is only if the owner chooses to sell his land, and the
builder or planter fails to purchase it where its value is not more than the value of the improvements, that the owner may remove the improvements from the
land. The owner is entitled to such remotion only when, after having chosen to sell his land, the other party fails to pay for the same.

As to the liability of Vergon, petitioners failed to present sufficient evidence to show negligence on Vergons part. Petitioners claim is obviously one (1) for tort,
governed by Article 2176 of the Civil Code.

[G.R. No. L-2209. January 2, 1907. ]

SEGUNDO JAVIER, Plaintiff-Appellant, v. LONGINOS JAVIER, administrator of the estate of Manuel Javier and Perfecta Tagle, Defendant-Appellee.

FACTS:

 This case relates to the ownership of the lot, and of the house standing thereon, No. 521 Calle Real, Malate, Manila. The court below found that the
land belonged to the plaintiff as administrator of the estate of his father, Manuel Javier, and that the defendant Isabel Hernandez and Manuel
Ramon Javier, her son, are the owners of the house standing on the lot. Judgment was rendered in favor of the plaintiff for the possession of the
property, but giving the defendants a reasonable opportunity to remove the house.
 The evidence sustains the findings of fact to the effect that the land belongs to the estate represented by the plaintiff. There was evidence to show
that the land was, in 1860, in the possession of Manuel Javier, the father of the defendant Segundo Javier, and that since that time it has been
occupied by his children and that no one of these children ever made any claim to the ownership thereof, and no one them ever occupied the
property as owner.
 Manuel Ramon Javier, testifying as a witness, made no claim to the ownership of the land, and testified simply that the result of his investigations
into the question of ownership showed that there was a great confusion in regard thereto.

11
 The appellants claim that this action can not be maintained by the administrator of the estate of Manuel Javier, but that it should be maintained by
all the heirs of the deceased. The right of judicial administrator to recover the possession of real property belonging to the estate of the deceased
was recognized in the case of Alfonso vs.Natividad
 The appellants claim in their brief that they were possessors in good faith, and by reason thereof and of the provisions of article 451 of
the Civil Code they can not be compelled to pay rent.
 It is also claimed by the appellants that, in accordance with article 453 of the Civil Code, they are entitled to be reimbursed for the expenses of
constructing the house

ISSUE: whether or not the appellants are possessors in good faith

RULING:

It is to be observed, however, that the appellants do not come within the definition of a possessor in good faith found in article 433 of the Civil
Code cited in their brief. As said by the appellants themselves in that brief, the two defendants, Segundo Javier and his wife, Isabel Hernandez,
always believed that the land did not belong to them but belonged to the estate of Manuel Javier. It is to be observed, moreover, that the judgment
of the court does not allow any recovery at all for the use or occupation of the house, and the recovery of rent for the use of the land is limited to
the time elapsed since April 24, 1904, when a demand was made upon the defendants for the possession of the property.

As to the reimbursement entitlement, the expenses are only allowed in accordance with the article cited by the appellants to a possession in good faith, and
the appellants were not such possessors. lawphil.net

Article 397 of the Civil Code relates to improvements made upon the common property by one of the coowners. The burden of proof was on the appellants to
show that the house was built with the consent of their cotenants. Even if a tacit consent was shown this would not require such cotenants to pay for the
house. (8 Manresa, Commentaries on Civil Code, p. 396.)

The judgment of the court below allowed the appellants to remove the house within a reasonable time. Whether this judgment was erroneous as far as the
appellee is concerned, we need not inquire, because he has not appealed from the judgment.

The judgment of the court below is affirmed, with the costs of this instance against the appellants.

LEONOR GRANA and JULIETA TORRALBA, petitioners,


vs.
THE COURT OF APPEALS, AURORA BONGATO and JARDENIO SANCHEZ, respondents.

G.R. No. L-12486 August 31, 1960

FACTS:
 Leonor Grana and Julieta Torralba were sued by Aurora Bongato and Jardenio Sanchez, for the recovery of 87 square meters of residential land.
 After trial, the court rendered judgment declaring the plaintiffs, herein respondents, owners of the land in controversy and ordering petitioners to
vacate and deliver it to said respondents and to pay a monthly rental of P10.00 from the filing of the complaint until they actually vacate the same.
 The record discloses that sometime in 1909 a cadastral survey of Butuan, Agusan, was made by the Bureau of Lands. In that survey, the parcel of
land here in question was included as part of the lot belonging to Gregorio Bongato and Clara Botcon
 this lot was purchased by the spouses Marcos Bongato and Eusebia More, and upon their death, the land was inherited by the respondents Aurora
Bongato and Jardenio Sanchez, the former being the daughter of Marcos Bongato by his first marriage while the latter is the son of Eusebia More
also by her first marriage.
 Petitioners claim that the lands in Butuan were subsequently resurveyed due to conflicts and overlapping of boundaries.
 Petitioners claim that the lands in Butuan were subsequently resurveyed due to conflicts and overlapping of boundaries. In that resurvey, Gregorio
Bongato's lot, according to petitioners, was identified as Lot No. 311 and that of Isidaria Trillo, their predecessor in interest, as Lot No. 310.
 Citing the fact that Original Certificate of Title No. RO-72 (138) covers 295 square meters of land, while the sketch plan of the second cadastral
survey of Butuan shows that Lot No. 311 has only 230 square meters, petitioners maintain that it is the latter area properly belongs to
respondents and that the land in question is part of the adjoining land, Lot No. 310, which belonged to their predecessor in interest.

ISSUE: Whether the CA erred in its ruling

RULING:
Although without any legal and valid claim over the land in question, petitioners, however, were found by the Court of Appeals to have constructed a portion of
their house thereon in good faith. Under Article 361 of the old Civil Code (Article 448 of the new), the owner of the land on which anything has been built
in good faith shall have the right to appropriate as his own faith shall have the right to appropriate as his own the building, after payment to the

12
builder of necessary and useful expenses, and in the proper case, expenses for pure luxury or mere pleasure, or to oblige the builder to pay the
price of the land.

Respondents, as owners of the land, have therefore the choice of either appropriating the portion of petitioners' house which is on their land upon
payment of the proper indemnity to petitioners, or selling to petitioners that part of their land on which stands the improvement. It may here be
pointed out that it would be impractical for respondents to choose to exercise the first alternative, i.e., buy that portion of the house standing on their land, for
in that event the whole building might be rendered useless.

The more workable solution, it would seem, is for respondents to sell to petitioners that part of their land on which was constructed a portion of the latter's
house. If petitioners are unwilling or unable to buy, then they must vacate the land and must pay rentals until they do so. Of course, respondents cannot oblige
petitioners to buy the land if its value is considerably more than that of the aforementioned portion of the house. If such be the case, then petitioners must
pay reasonable rent. The parties must come to an agreement as to the conditions of the lease, and should they fail to do so, then the court shall fix
the same. (Article 361, old Civil Code; Article 448 of the new).

In this connection, the appellate court erred in ordering petitioners to pay monthly rentals of P10.00 from the date of filing of the complaint until they
actually vacate said land. A builder in good faith may not be required to pay rentals. He has a right to retain the land on which he has built in good
faith until he is reimbursed the expenses incurred by him.

DAMIAN IGNACIO, FRANCISCO IGNACIO and LUIS IGNACIO, petitioners, vs. ELIAS HILARIO and his wife DIONISIA DRES, and FELIPE NATIVIDAD,
Judge of First Instance of Pangasinan, respondents.

FACTS:

 Elias Hilario and his wife Dionisia Dres are the owners of the whole property of a parcel of land, partly rice-land and partly residential.
 Damian, Francisco, and Luis (all surnamed Ignacio) built some buildings therein (a granary and a house)
 After trial in the Court of First Instance of Pangasinan presided over by Hon. Alfonso Felix , he rendered judgment holding spouses Hilario as the
legal owners of the whole property but conceding to defendants the ownership of the houses and granaries built by them but conceding to Ignacios
the ownership of the houses and granaries built by them with rights of a possessor in good faith, in accordance with Article 361 of the Civil Code.
That the Igancios are entitled to hold the possession of the residential lot after they are paid the actual market value of their houses and granaries
erected thereon unless the plaintiffs prefer to sell them said residential lot, in which case defendants shall pay the plaintiffs the proportionate value
of said residential lot.
 A motion was filed in the same CFI now presided by Judge Felipe Natividad
 Spouses pray for am order of execution alleging that since they chose neither to pay the Ignacios for the buildings nor to sell to them the residential
lot, Igancios should be ordered to remove the structure at their own expense and to restore to spouses in the possession of said lot.
 Judge Natividad granted “Ignacios” objection

ISSUE: Whether or not Hilario, the owner in good faith, may eject a builder in good faith without choosing either to appropriate the building for himself after
payment of its value or to sell his land to the builder in good faith.

RULING: The owner of the building evicted in good faith on a land owned by another, is
entitled to retain the possession of the land until he is paid. the value of his building, under article 453 of the Civil Code. The owner of the land,
upon the other hand, has the option, under article 361, either to pay for the building or to sell his land to the owner of the building.
But he cannot, as respondents here did, refuse both to pay for the building and to sell the land and compel the owner of the building to remove it
from the land where it is erected. He is entitled to such remotion only when, after having chosen to sell his land, the other party fails to pay for the
same.

The SC,hold therefore, that the order of Judge Natividad compelling defendants-petitioners to remove their buildings from the land belonging to
plaintiffs-respondents only because the latter chose neither to pay for such buildings nor to sell the land, is null and void.

BELEN UY TAYAG and JESUS B. TAYAG, petitioners, vs. ROSARIO YUSECO, JOAQUIN C. YUSECO and THE COURT OF APPEALS, respondents.

FACTS

 In 1920’s, Maria Lim offered to Joaquin Yuseco and his wife, defendant Rosario Yuseco,
for them to build a house on 2 lots owned by Lim (Hacienda de San Lazaro).
 The Yusecos accepted the offer and built a dwelling house and an annex for garage and servant quarters.
 To legalize, Lim and the Yusecos executed a contract of lease covering them, to run for a period of 5 years, with a yearly rental of P120.
 Before death, Lim sold the 2 lots to her married daughter, Belen Tayag.
 In 1946, the Tayags asked the Yusecos to remove their house, or else pay them monthly rent. The latter refused and because of this, the Tayags
brought an action of ejection for the restitution of the lots and the recovery of a monthly rental.
 Judgment was rendered in favor of the Tayags. The defendants appealed.

13
 The court rendered judgment adjudicating to Tayag the possession of the 2 lots, with right to appropriate the 2 buildings thereon upon payment to
Yusecos of their value, and providing that should the plaintiffs fail to pay within 90 days, the defendants shall have the right to purchase said lots.
The plaintiffs appealed.
 CA, finding the defendants as possessors in good faith, affirmed the lower court. The plaintiffs appealed to SC.
 SC affirmed the decision of CA to remand the case to lower court to require plaintiffs to make a choice: "whether they would purchase the buildings
erected on the lots, or allow the defendants to buy said lots.”
 Tayag’s choice: “NOT to sell the lot, but to appropriate the building if its valuation is as it should be and is properly and fairly determined.”
 Trial court ordered the next step to adduce evidence in connection with the value of the buildings. Since the defendants will be the sellers, they
have the burden of proving the value of the buildings, giving the plaintiffs opportunity to rebut.

ISSUE: WHETHER OR NOT TAYAG MAY STILL REPUDIATE HIS ELECTION TO APPROPRIATE THE BUILDINGS.

HELD:

NO. Once a choice is made by the landowner, it is generally irrevocable.Thus, if the landowner has opted/elected to appropriate the building but he
is unable to pay for the indemnity or amount, the landowner CANNOT afterwards opt/elect to sell the land. Since Tayag’s first choice had already
been communicated to the court and she had already been ordered to pay, her duty has been converted into a monetary obligation which can be
enforced by a writ of execution.

Art. 448 provides “The owner of the land on which anything has been built, sown or planted in good faith, shall have the right to appropriate as his own the
works, sowing or planting, after payment of the indemnity provided for in Articles 546 and 548, or to oblige the one who built or planted to pay the price of the
land, and the one who sowed, the proper rent. However, the builder or planter cannot be obliged to buy the land if its value is considerably more than that of
the building or trees. In such case, he shall pay reasonable rent, if the owner of the land does not choose to appropriate the building or trees after proper
indemnity. The parties shall agree upon the terms of the lease and in case of disagreement, the court shall fix the terms thereof. (361a)”

SPOUSES CONCEPCION FERNANDEZ DEL CAMPO and ESTANISLAO DEL CAMPO, plaintiffs-appellees, vs. BERNARDA FERNANDEZ ABESIA,
defendant-appellant.

Facts:

 The case involves a parcel of land, situated at the corner of F. Flores and Cavan Streets, Cebu City.
 An action for partition was filed by plaintiffs Sps Del Campo in the CFI of Cebu.
 Plaintiffs and defendants are co-owners pro indiviso of this lot in the proportion of and 1/3 share each, respectively.
 The trial court appointed a commissioner in accordance with the agreement of the parties. The said commissioner conducted a survey, prepared a
sketch plan and submitted a report to the trial court on May 29, 1976, recommending that the property be divided into two lots: Lot 1161-A with an
area of 30 square meters for plaintiffs and Lot No. 1161-B with an area of 15 square meters for the defendants.
 The houses of plaintiffs and defendants were surveyed and shown on the sketch plan. The house of defendants occupied the portion with an area
of 5 square meters of Lot 1161-A of plaintiffs.
 The parties manifested their conformity to the report and asked the trial court to finally settle and adjudicate who among the parties should take
possession of the 5 square meters of the land in question.

ISSUE:

Whether or Not Article 448 of the Civil Code is applicable to a builder in good faith when the property involved is owned in common.

Held: When the co-ownership is terminated by the partition and it appears that the house of defendants overlaps or occupies a portion of 5 square
meters of the land pertaining to plaintiffs which the defendants obviously built in good faith, then the provisions of Article 448 of the new Civil
Code should apply. Manresa and Navarro Amandi agree that the said provision of the Civil Code may apply even when there was co-ownership if
good faith has been established.

Applying the aforesaid provision of the Civil Code, the plaintiffs have the right to appropriate said portion of the house of defendants upon
payment of indemnity to defendants as provided for in Article 546 of the Civil Code. Otherwise, the plaintiffs may oblige the defendants to pay the
price of the land occupied by their house. However, if the price asked for is considerably much more than the value of the portion of the house of
defendants built thereon, then the latter cannot be obliged to buy the land. The defendants shall then pay the reasonable rent to the plaintiff upon
such terms and conditions that they may agree. In case of disagreement, the trial court shall fix the terms thereof. Of course, defendants may
demolish or remove the said portion of their house, at their own expense, if they so decide.

Article 448 of the New Civil Code provides as follows:

Art. 448. The owner of the land on which anything has been built, sown, or planted in good faith, shall have the right to appropriate as his own the works,
sowing or planting, after payment of the indemnity provided for in articles 546 and 548, or to oblige the one who built or planted to pay the price of the land,

14
and the one who sowed, the proper rent. However, the builder or planter cannot be obliged to buy the land if its value is considerably more than that of the
building or trees. In such case, he shall pay reasonable rent, if the owner of the land does not choose to appropriate the building or trees after proper
indemnity. The parties shall agree upon the terms of the lease and in case of disagreement, the court shall fix the terms thereof.

PEDRO P. PECSON, petitioner, vs. COURT OF APPEALS, SPOUSES JUAN NUGUID and ERLINDA NUGUID, respondents.

FACTS:

 Pedro Pecson was the owner of a commercial lot located in Kamias Street, Quezon City on which he built a 4-door-2-storey apartment building.
 He failed to pay realty taxes amounting to P12k so the lot was sold at public auction to Mamerto Nepomuceno who later on sold it to the Sps.
Nuguid on 12 October 1983 for 103,000 php.
 Pecson challenged the validity of the auction before the RTC but was dismissed but the RTC held that the apartment building was not subject of the
litigation.
 On appeal, the CA affirmed in toto the decision of the RTC that the apartment bldg was not included in the auction sale - Evincing that it was only
the land without the apartment building which was sold at the auction sale. Furthermore, Property subject of the auction sale at which Mamerto
Nepomuceno was the purchaser is referred to as Lot No. 21-A, Block No. K-34, at Kamias, Barangay Piñahan, with an area of 256.3 sq.m., with no
mention whatsoever, of the building thereon.
 After an entry of judgment was made, the Sps. Nuguid filed a motion with the RTC for a motion for delivery of possession of the lot and the
apartment building citing Art. 546 of the CC. The RTC issued an order declaring that the owner of the lot and apartment bldg were the Sps. Nuguid
and to pay the construction cost of the apartment before a writ of possession would be issued and to pay rent to the spouses.
 Pecson moved for reconsideration but the Trial court did not act on it, instead it issued a writ of possession.
 The CA affirmed in part the decision declaring the cost of construction can be offset from the amount of rents to be collected and that since Sps.
Nuguid opted to appropriate the improvement, Pecson is entitled to be reimbursed the cost of construction at the time it was built in 1965 which is
at P53k and the right to retain the improvement until full indemnity is paid. - Thus the case at bar.

ISSUE
WON the Nuguids should reimburse Pecson for the benefits derived from the apartment building

HELD : YES.

Since petitioners opted to appropriate the improvement for themselves as early as June 1993, when they applied for a writ of execution despite knowledge that
the auction sale did not include the apartment building, they could not benefit from the lot’s improvement, until they reimbursed the improver in full, based on
the current market value of the property.

Under Article 448, the landowner is given the option, either to appropriate the improvement as his own upon payment of the proper amount of
indemnity or to sell the land to the possessor in good faith. Relatedly, Article 546 provides that a builder in good faith is entitled to full
reimbursement for all the necessary and useful expenses incurred; it also gives him right of retention until full reimbursement is made.

The right of retention is considered as one of the measures devised by the law for the protection of builders in good faith. Its object is to
guarantee full and prompt reimbursement as it permits the actual possessor to remain in possession while he has not been reimbursed (by the
person who defeated him in the case for possession of the property) for those necessary expenses and useful improvements made by him on the
thing possessed.

Given the circumstances of the instant case where the builder in good faith has been clearly denied his right of retention for almost half a decade, we find that
the increased award of rentals by the RTC was reasonable and equitable. The petitioners had reaped all the benefits from the improvement introduced by the
respondent during said period, without paying any amount to the latter as reimbursement for his construction costs and expenses. They should account and
pay for such benefits.

SULO SA NAYON, INC. and/or PHILIPPINE VILLAGE HOTEL, INC. and JOSE MARCEL E. PANLILIO, petitioners, vs. NAYONG PILIPINO
FOUNDATION,
respondent.

FACTS:

 In 1975, respondent leased a portion of the Nayong Pilipino Complex, to petitioner Sulo sa Nayon, Inc. for the construction and operation of a hotel
building, to be known as the Philippine Village Hotel. The lease was for an initial period of 21 years, or until May 1996. It is renewable for a period
of 25 years under the same terms and conditions upon due notice in writing to respondent of the intention to renew.

15
 In 1995, petitioners sent respondent a letter notifying the latter of their intention to renew the contract for another. July of the same year, parties
agreed to the renewal of the contract for another 25 years, or until 2021. Under the new agreement, petitioner PVHI was bound to pay the monthly
rentals

 Beginning January 2001, petitioners defaulted in the payment of their monthly rental. Respondent repeatedly demanded petitioners to pay the
arrears and vacate the premises.

 MeTC rendered its decision in favor of respondent


 RTC which modified the ruling of the MeTC.
 CA which held that the RTC erroneously applied the rules on accession, as found in Articles 448 and 546 of the Civil Code

ISSUE:

WON Sulo sa Nayon as builders have acted in good faith in order for Art. 448 in relation to Art. 546 of the Civil Code may apply with respect to their rights over
improvements.

HELD:

Introduction of valuable improvements on the leased premises does not give the lessee the
right of retention and reimbursement which rightfully belongs to a builder in good faith—the doctrine is that a lessee is neither a builder in good
faith nor in bad faith that would call for the application of Articles 448 and 546 of the Civil Code since his rights are governed by Article 1678.

Article 448 is manifestly intended to apply only to a case where one builds, plants, or sows on land in which he believes himself to have a claim of title, and not
to lands where the only interest of the builder, planter or sower is that of a holder, such as a tenant.

In the case at bar, petitioners have no adverse claim or title to the land. In fact, as lessees, they recognize that the respondent is the owner of the
land. What petitioners insist is that because of the improvements, which are of substantial value, that they have introduced on the leased premises
with the permission of respondent, they should be considered builders in good faith who have the right to retain possession of the property until
reimbursement by respondent.

We affirm the ruling of the CA that introduction of valuable improvements on the leased premises does not give the petitioners the right of retention and
reimbursement which rightfully belongs to a builder in good faith. Otherwise, such a situation would allow the lessee to easily "improve" the lessor out of its
property. We reiterate the doctrine that a lessee is neither a builder in good faith nor in bad faith that would call for the application of Articles 448 and 546 of
the Civil Code. His rights are governed by Article 1678 of the Civil Code.

G. R. No. 164489

December 14, 2005

FERNANDO CARRASCOSO, JR., Petitioner,

-versus-

THE HONORABLE COURT OF APPEALS, LAURO LEVISTE, as Director and Minority Stockholder and On Behalf of Other Stockholders of El Dorado
Plantation, Inc. and EL DORADO PLANTATION, INC., represented by one of its minority stockholders, Lauro P. Leviste, Respondents.

FACTS:

 In March 1972, El Dorado Plantation, Inc. (El Dorado), through its board member Lauro Leviste, executed a Deed of Sale with Fernando
Carrascoso, Jr. The subject of the sale was a 1,825 hectare of land located in Sablayan, Occidental Mindoro. It was agreed that Carrascoso was to
pay P1.8M.; that P290K would be paid by Carrascoso to PNB to settle the mortgage upon the said land. P210k would be paid directly to Leviste.
The balance of P1.3M plus 10% interest would be paid over the next 3 years at P519k every 25th of March. Leviste also assured that there were no
tenants hence the land does not fall under the Land Reform Code. Leviste allowed Carrascoso to mortgage the land which the latter did.
 Carrascoso obtained a total of P1.07M as mortgage and he used the same to pay the down payment agreed upon in the contract. Carrascoso
defaulted from his obligation which was supposed to be settled on March 25, 1975. Leviste then sent him letters to make good his end of the
contract otherwise he will be litigated. Lauro and El Dorado sought the cancellation of the Transfer of Certificate of Title in the name of Carrascoso.
 On July 11, 1975, Carrascoso and the Philippine Long Distance Telephone Company (PLDT), through its President Ramon Cojuangco, executed
an Agreement to Buy and Sell whereby the former agreed to sell 1,000 hectares of the property to the latter at a consideration of P3,000.00 per
hectare or a total of P3,000,000.00. The July 11, 1975 Agreement to Buy and Sell was not registered and annotated on Carrascosos TCT.

16
 On March 15, 1977, Lauro and El Dorado finally filed on March 15, 197 a complaint for rescission of the 1972 Deed of Sale of Real Property
between El Dorado and Carrascoso Lauro and El Dorado and caused the annotation on the Transfer of Certificate Titlle of the subject property a
Notice of Lis Pendens.
 Lis Pendens - Lis pendens literally means a pending suit. Doctrine that refers to the jurisdiction, power or control which a court acquires over a
property involved in a suit, pending the continuance of the action, until final judgment.
 In 1977, Carrascoso executed a Deed of Absolute Sale of the subject property of their July 11, 1975 Agreement to Buy and Sell with PLDT. The
subject of the sale was the same land sold to Carrascoso by Leviste but it was only the 1000 sq. m. portion thereof. The land is to be sold at P3M.
Part of the terms and conditions agreed upon was that Carrascoso is to remove all tenants from the land within one year. He was also being given
a 6-month extension in case he’ll need one. Thereafter, PLDT will notify Carrascoso if whether or not PLDt will finalize the sale. Eventually, PLDT
gained possession of the land and itroduced improvements on the land.
 Meanwhile, El Dorado filed a civil case against Carrascoso. PLDT intervened averring that it was a buyer in good faith that they were not aware of
any litigation involving the 1,000 hectare portion of the property or of any flaw in his title and when PLDT executed the Deed of Absolute Sale they
had no knowledge of any pending litigation over the property and neither were they aware that a notice of lis pendens had been annotated on
Carrascosos title. The Regional Trial Court (RTC) ruled in favor of Carrascoso. The Court of Appeals (CA) reversed the RTC ruling.

ISSUE:

Whether or not PLDT acted in bad faith in purchasing the contested property.

RULING:

 No. The SC ruled that, PLDT commenced construction of improvements on the 1,000 hectare portion of the property immediately after the
execution of the July 11, 1975 Agreement to Buy and Sell with the full consent of Carrascoso. Thus, until March 15, 1977 when the Notice of Lis
Pendens was annotated on the property, PLDT is deemed to have been in good faith in introducing improvements on the 1,000 hectare portion of
the property.
 As regards the improvements introduced by PLDT on the 1,000 hectare portion of the property, a distinction should be made between those which
it built prior to the annotation of the notice of lis pendens and those which it introduced subsequent thereto.
 After March 15, 1977, however, PLDT could no longer invoke the rights of a builder in good faith.
 Should El Dorado then opt to appropriate the improvements made by PLDT on the 1,000 hectare portion of the property, it should only be made to
pay for those improvements at the time good faith existed on the part of PLDT or until March 15, 1977, to be pegged at its current fair market value.
 The commencement of PLDTs payment of reasonable rent should start on March 15, 1977 as well, to be paid until such time that the possession of
the 1,000 hectare portion is delivered to El Dorado, subject to the reimbursement of expenses as aforestated, that is, if El Dorado opts to
appropriate the improvements.
 If El Dorado opts for compulsory sale, however, the payment of rent should continue up to the actual transfer of ownership.
 When a person builds in good faith on the land of another, Article 448 of the Civil Code governs: Art. 448. The owner of the land on which anything
has been built, sown or planted in good faith, shall have the right to appropriate as his own the works, sowing or planting, after payment of the
indemnity provided for in Articles 546 and 548, or to oblige the one who built or planted to pay the price of the land, and the one who sowed, the
proper rent. However, the builder or planter cannot be obliged to buy the land if its value is considerably more than that of the building or trees. In
such a case, he shall pay reasonable rent, if the owner of the land does not choose to appropriate the building or trees after the proper indemnity.
The parties shall agree upon the terms of the lease and in case of disagreement, the court shall fix the terms thereof. The above provision covers
cases in which the builders, sowers or planters believe themselves to be owners of the land or, at least, to have a claim of title thereto. Good faith is
thus identified by the belief that the land is owned; or that by some title one has the right to build, plant, or sow thereon.
 When a person builds in bad faith on the land of another, Articles 449 and 450 govern: Art. 449. He who builds, plants or sows in bad faith on the
land of another, loses what is built, planted or sown without right to indemnity. Art. 450. The owner of the land on which anything has been built,
planted or sown in bad faith may demand the demolition of the work, or that the planting or sowing be removed, in order to replace things in their
former condition at the expense of the person who built, planted or sowed; or he may compel the builder or planter to pay the price of the land, and
the sower the proper rent.

G.R. No. 178906 February 18, 2009

ELVIRA T. ARANGOTE, petitioner,

-versus-

SPS. MARTIN MAGLUNOB and LOURDES S. MAGLUNOB, and ROMEO SALIDO, Respondents.

17
FACTS:

 Elvira Arangote acquired the subject parcel of land from Esperanza Maglunob, who is grand aunt of respondents Martin Maglunob and Romeo
Salido. In June 1986, Esperenza executed an affidavit in which she renounced her rights, share and participation in the land in favor of Elvira and
her husband. It appears that the lot was not exclusive property of Esperanza but also of the other heirs of Martin I whom she represented in the
partition agreement.
 Elvira and her husband, Ray constructed a house on the land in 1989 and in 1993, OCT was issued in her name by the DAR. However,
respondents with the help of hired persons entered the property and built a wall behind and in front of Elvira’s house. Elvira and Ray sued
respondents for quieting of title and declaration of ownership. Respondents averred that they were co-owners of the land with Esperanza who
allegedly inherited the land from Martin together with Tomas and Inocencia (Martin 2’s and Romeo’s predecessor in interest).
 They argued that Esperanza could not have validlywaived her rights in favor of Elvira and Ray. MCTC ruled for Elvira. RTC reversed MCTC and
declared respondents’ lawful owners of the land together with the other heirs of Martin I. Elvira went to the CA but the CA affirmed the RTC
decision. Before SC, Elvira argued that both RTC and CA erred in declaring the affidavit of Esperanza void because it is a valid and binding proof of
transfer of the subject property as it was coupled with actual delivery.

ISSUE:

Whether or not the donation to Elvira and her husband is valid.

HELD:

 Supreme Court affirmed the decision of CA. SC ruled that the affidavitexecuted by Esperanza wherein she renounced, relinquished and waivedall
her rights, share, interest and participation in the subject property infavor of Elvira and Ray is in fact a donation. Thus, it should have complied with
therequirements of Article 749 of the Civil Code.
 A simple donation of real property to be valid

1) Must be made in a public instrument;

2) It must be accepted, which must be in the same deed of donation or in a separate public instrument;

3) If the acceptance is made in a separate instrument, the donor must be notified inauthentic form and the same must be noted in both instruments.

 The affidavit executed by Esperanza relinquishing her rights, share and participation over the property in favor of Elvira suffered from legal
infirmities. In the present case, the said Affidavit, which is tantamount to a Deed of Donation, met the first requisite, as it was notarized; thus, it
became a public instrument.
 Nevertheless, it failed to meet the aforesaid second and third requisites. The acceptance of the said donation was not made by the petitioner and
her husband either in the same Affidavit or in a separate public instrument. As there was no acceptance made of the said donation, there was also
no notice of the said acceptance given to the donor, Esperanza. Therefore, the Affidavit executed by Esperanza in favor of petitioner and her
husband is null and void.

[G.R. No. 136456. October 24, 2000]

HEIRS OF RAMON DURANO, SR., RAMON DURANO III, AND ELIZABETHHOTCHKISS DURANO, petitioners,

-versus-

SPOUSES ANGELES SEPULVEDA UY AND EMIGDIO BING SING UY, SPOUSES FAUSTINO ALATAN AND VALERIANA GARRO, AURELIA MATA,
SILVESTRE RAMOS, HERMOGENES TITO, TEOTIMO GONZALES, PRIMITIVA GARRO, JULIAN GARRO, ISMAEL GARRO, BIENVENIDO CASTRO,
GLICERIO BARRIGA, BEATRIZ CALZADA, ANDREA MATA DE BATULAN, TEOFISTA ALCALA, FILEMON LAVADOR, CANDELARIO LUMANTAO,
GAVINO QUIMBO, JUSTINO TITO, MARCELINO GONZALES, SALVADOR DAYDAY, VENANCIA REPASO, LEODEGARIO GONZALES, and
RESTITUTA GONZALES, respondents.

FACTS:

 Congressman Ramon Durano, Sr, and son Ramon Durano III and the latter’s wife Elizabeth Hotchkiss Durano (herein petitioners) filed a case for
damages against respondents for allegedly officiating a “hate campaign” against them by lodging complaints for invasion of the respondents’
properties in Cahumayhumayan, Danao City. The complaints were filed with the Police Department of Danao and the Office of the President. The

18
said complaints were investigated by the Department of Justice through the City Fiscal and the Philippine Constabulary who later on dismissed the
complaints for being baseless. The petitioners added that the respondents spread false rumours and tales which subjected them to public contempt
and ridicule.
 The respondents made a counterclaim demanding the return of their properties claiming that in August 1970, they received mimeographed notices
signed by Durano Sr. informing them that the land they occupied is owned by Cebu Portland Cement Company and was purchased by Durano &
Co for immediate turn over. However, before many of them could even receive the notice, employees of Durano & Co. proceeded to bulldoze the
land, destroying plantings and improvements made therein. On September 15, 1970, Durano & Co. sold the subject land to Durano III. Claiming
that during that time, they were not able to find local relief as Durano Sr.’s wife was the Mayor at that time causing them to send a letter to then
President Marcos.
 On April 22, 1975, petitioners moved to dismiss their own complaint – granted by the RTC without prejudice to the counterclaim of the respondents.
According to the petitioners, the property originally belonged to Cepoc and was sold to Durano & Co., and later on to Durano III. But Durano III
claimed that he only learned of the bulldozing when complaints were already filed by the respondents. He further claimed that they dismissed the
complaints against the respondents as a form of reconciliation with them but the latter still pursued their counterclaim. According to him, the
properties of the claimants, except for Sepulveda Uy, daughter of former Mayor of Danao, were occupants of the said property and Durano & Co.
purchased the adjacent property for mining coal.
 The RTC ordered in their ruling that the petitioners are to pay damages to the respondent and the return of the properties of Venancia Repaso,
Hermogenes Tito, and Marcelino Gonzales as well as the property of Angeles Sepulveda Uy with respect to the are found outside of the Cepoc
property. On appeal, the CA affirmed the decision but modified the judgement ordering the return of all properties to the respondents.

ISSUE:

Whether or not the respondents are builders in good faith.

RULING:

 The court ruled that the records indicated that the respondents’ possession has already ripened into ownership by acquisitive prescription.
Acquisitive prescription is acquired by possession in good faith with just tittle for a period of ten years. One is considered in good faith when he is
not aware of any flaw in his tittle or mode of acquisition of the property and there is just title when the adverse claimant came into possession of the
property through one of the modes of acquiring ownership provided by law.
 In the case at bar, the respondents acquired the properties by purchase or inheritance and ever since were in actual, continuous, open, and
adverse possession. The records showed that they were unaware of any claims over the properties until the notices given on August 1970.
 The petitioners on the other hand cannot claim good faith. The validity of the certificates of title obtained by them were doubted by the courts as
there was a lack of registered title of Cepoc and the deed of sale between Cepoc and Durano & Co. were not notarised and therefore
unregisterable. Furthermore, a buyer could not have been ignorant that the property they bought were adversely possessed by the respondents nor
did they investigate the property – the petitioners cannot be held to be buyers in good faith, nor builders in good faith.
 Under the Article 449 of the New Civil Code, he who builds etc. in bad faith on the land of another, loses what is built etc. without right of indemnity.
Furthermore, Article 450 gives the landowner over which something was built in bad faith the power to demolish the works to replace the property in
their former condition at the expense of the builder. And Article 451 gives him the right to damages.

G.R. No. L-17681 February 26, 1965

MINDANAO ACADEMY, INC., MAURICIO O. BAS, ERLINDA D. DIAZ, accompanied by her husband ANTOLIN DIAZ, ESTER AIDA D. BAS,
accompanied by her husband MAURICIO O. BAS, ROSALINDA D. BELLEZA, accompanied by her husband APOLINARIO BELLEZA, LUZ MINDA D.
DAJAO, accompanied by her husband ELIGIO C. DAJAO, ADELAIDA D. NUESA, accompanied by her husband WILSON NUESA, PEDRO N.
ABUTON, SY PAOCO, JOSEFA DIGNUM, and PERFECTO VELASQUEZ, plaintiffs-appellees,

-versus-

ILDEFONSO D. YAP, ROSENDA A. DE NUQUI, and SOTERO A. DIONISIO, JR., defendants,

ILDEFONSO D. YAP, defendant-appellant.

Facts:

 Rosenda A. de Nuqui and her son Sotero Dionisio, Jr. sold 4 parcels of land (3 residential lands in Oroquieta, Misamis Occidental and 1 in
Ozamis City) to Ildefonso D. Yap via a Mutual Agreement dated May 10, 1964

19
* Another daughter, Adelaida Dionisio-Nuesa, was named as co-owner in the deed of sale, however did not actually take part thereof.

 Situated on the parcels of land were certain buildings as well as laboratory equipment, books, furniture and fixtures used by two schools. These
were also included in the sale to Yap. The two schools (both owned by Mindanao Academy, Inc.) are:

a. Mindanao Academy - in Oroquieta, Misamis Occidental


b. Misamis Academy - in Ozamis

 However, there were other co-owners of the properties namely (not included in the Mutual Agreement):

a. Children of Rosenda & deceased husband Sotero Dionisio, Sr.: Erlinda D. Diaz, Ester Aida D. Bas, Rosalinda D. Belleza and Luz
Minda D. Dajao (solely for the lands)

b. Mindanao Academy, Inc. (with regards to school buildings, equipment, books, furniture and fixtures, including a laboratory situated in
the lands)

 Yap took over the properties and renamed the schools to Harvardian Colleges.
 On July 4, 1954, Erlinda Diaz filed action to assert her rights as co-owner.
 On July 31, 1956, Yap filed his first pleading.

CFI LEVEL:

Two actions were initiated:

a. Action for annulment of the sale and recovery of rents and damages - filed by Mindanao Academy, some stockholders, and the 5 children of
Nuqui named above VS. Nuqui and son Dionisio

Held:

i. Yap is hereby ordered to pay costs


ii. Mutual Agreement is null and void
iii. Yap ordered to restore all the buildings and grounds
iv. Yap is ordered to restore to the Mindanao Academy, all the books, laboratory apparatus, furniture and other equipment
described in the Mutual Agreement
v. Yap to return all the Records of the Mindanao Academy and Misamis Academy
vi. Yap to pay to the plaintiffs stockholders of the Mindanao Academy, Inc., the amount of P10,000.00 as nominal damages;
P3.000.00 as exemplary damages; and P2.000.00 as attorney's fees

b. Action for rescission of deed of sale – filed by Mother Nuqui, Dionisio, Jr. and Erlinda D. Diaz (and husband Antolin Diaz) VS. Yap

* The other 4 children did not join in the 2nd action as they already relinquished their claim to sister Erlinda Diaz (quitclaim)

Held:

i. Yap is hereby ordered to pay cost


ii. Mutual Agreement is null and void
iii. Yap ordered to restore all the buildings and grounds described in the Mutual Agreement together with all the permanent
improvements
iv. Yap ordered to pay to the plaintiffs the amount of P300.00 monthly from July 31, 1956 up to the time he shall have
surrendered the properties in question to the plaintiffs therein, plus P1,000.00 as attorney's fees to plaintiffs Antolin and
Erlinda D. Diaz

ISSUES:

1. Whether or not CFI erred in declaring Mutual Agreement null and void.
2. Whether or not CFI erred in holding him liable for rents.
3. Whether or not CFI erred in holding plaintiffs entitled to recover all the lands, buildings, and other permanent improvements described in
the Mutual Agreement.

RULING:

20
 Negative on all issues. Builder in bad faith loses right to be paid indemnity for what he built as just punishment for bad faith.
 Art. 449 of the NCC provides that he who builds, plants, or sows in bad faith on the land o f another, loses what is built, planted or sown without
right to indemnity.

I. Yap argues CFI erred in declaring Mutual Agreement null and void.

 SC: No error. The prestation involved were indivisible because the vendors (Nuqui and Sotero were not the only owners of either the
lands (co-owned by other children) or the buildings, equipments, laboratory, etc. (owned by Mindanao Academy). As admitted by Yap,
he would not have entered into the transaction except to acquire all of the properties.

II. Yap argues CFI erred in holding him liable for rents

 SC: No error. Both vendor (Nuqui and son) and vendee (Yap) acted in bad faith. If both acted in bad faith, they must be treated as
having acted in good faith. This fictive good faith of Yap’s excuses him from rental fees up to July 31, 1956 (the date of his first pleading)
because the filing of his first pleading made it known that there were other co-owners, so his fictive good faith ceases.

III. Yap argues CFI erred in holding plaintiffs entitled to recover all the lands, buildings, and other permanent improvements described in the Mutual
Agreement

 SC:

* Reimbursement for improvements (new building worth P8,000 and toilet worth P800) – Not granted because these were constructed after the
action for annulment was commenced, thereby tainting his good faith.

* Reimbursement for equipment, books, furniture, fixtures brought in by him – Not granted because what was adjudged to be restored was what
was described in the Mutual Agreement (not what he brought in), so this is outside the scope of the judgment

G.R. No. 140528

December 7, 2011

MARIA TORBELA, represented by her heirs, Petitioners,

- versus -

SPOUSES ANDRES T. ROSARIO and LENA DUQUE-ROSARIO and BANCO FILIPINO SAVINGS AND MORTGAGE BANK, Respondents.

FACTS:

 The issue is over a parcel of land inherited by the Torbela siblings from their parents.
They executed a deed of absolute quitclaim over the property in favor of Dr. Rosario. Four days after, a TCT was issued in Dr. Rosario’s name
covering the property.
 Another deed of absolute quitclaim was subsequently executed twelve days after by Dr. Rosario acknowledging that he only borrowed the lot from
the Torbela siblings and was already returning the same. This deed was notarized but not immediately annotated.
 Dr. Rosario used the land as mortgage for a loan he obtain through DBP for P70,000.00. He used the proceeds of the loan to build a 4 storey
building which was initially used as a hospital but later converted into a commercial space. Part was leased to PT&T and the rest to Rosario ’s
sister who operated the Rose Inn Hotel and Restaurant.
 Dr. Rosario fully paid the loan from DBP and the mortgage was cancelled and ratified by a notary public. However, Dr. Rosario took another loan
from PNB. He later acquired a third loan from Banco Filipino and bought out the loan from PNB cancelling the mortgage with PNB. Rosario failed to
pay their loan in Banco Filipino and the property was extrajudicially foreclosed.
 Meanwhile, back in 1965, the Torbela siblings sought to register their ownership over the lot and to perfect their title but couldn’t because the title
was still with DBP. They showed as proof the deed of absolute quitclaim presented executed by Rosario himself. In 1986, they filed a civil case for
recovery of ownership and possession and damages. They tried to redeem the lot from Banco Filipino but failed. TCT was issued to Banco
FIilipino.
 The Torbela’s claim they have right over the rents of the building through accession because they are the land owners.

ISSUE: Who has right over the improvements made on the lot and the rents thereof.

21
RULING:
 According to Art. 440, the accessory follows the principal. Ownership of property gives the right by accession to everything which is produced
thereby, or which is incorporated or attached thereto, either naturally or artificially.
 However, in the case at bar, both Torbela siblings and Rosario are deemed in bad faith. The Torbelas knew Rosario built on the land and even
allowed him to use the land to obtain a loan from DBP. Rosario on the other hand consciously built on land he knew was not his. They both had
knowledge and did not oppose.
 Art. 453 states that when both parties are in bad faith, the case shall be treated as though both were in good faith thus the application of Art. 448.
 448 allows the Land Owner 2 options in the case at bar. Either indemnify Rosario and appropriate the lot to himself or ask Rosario to buy the lot or
the rent rate. This case was remanded to the RTC for the Torbelas to make such decision. The case was remanded to the RTC for further
proceedings to determine the facts essential to the proper application of Articles 448 and 546 of the Civil Code, particularly: (a) the present fair
market value of Lot; (b) the present fair market value of the improvements thereon; (c) the option of the Torbela siblings to appropriate the
improvements on the lot or require Dr. Rosario to purchase said lot; and (d) in the event that the Torbela siblings choose to require Dr. Rosario to
purchase the lot but the value thereof is considerably more than the improvements, then the reasonable rent of the lot to be paid by Dr. Rosario to
the Torbela siblings
 Still following the rules of accession, civil fruits such as rent belong to the owner of the building. Rosario has rights over the rent and improvements
and shall continue until the Torbela siblings have chosen an option from 448.

G.R. No. 162593 September 26, 2006


REMEGIA Y. FELICIANO, Substituted by the Heirs of REMEGIA Y. FELICIANO, as represented by NILO Y. FELICIANO, petitioners, vs. SPOUSES
AURELIO and LUZ ZALDIVAR, respondents.

FACTS:
 Feliciano filed against the spouses Aurelio and Luz Zaldivar a complaint for declaration of nullity of Transfer Certificate of Title and reconveyance of
the subject property in Cagayan de Oro City. The said title is registered in the name of Aurelio Zaldivar.
 In her complaint,Feliciano alleged that she was the registered owner of a parcel of land covered by a TCT. Sometime in 1974, Aurelio, allegedly
through fraud, was able to obtain a TCT covering the portion of Feliciano’s lot as described in her TCT.
 According to Feliciano, the subject lot was originally leased from her by Pio Dalman, Aurelio’s father-in-law. She further alleged that she was going
to mortgage the subject lot to Ignacio Gil which however, did not push through because Gil took back the money without returning the receipt she
had signed as evidence of the supposed mortgage contract. Thereafter, in 1974, Aurelio filed with the then CFI of Misamis Oriental a petition for
partial cancellation of theTCT in Feliciano’s name. It was allegedly made to appear therein that Aurelio and his spouse Luz acquired the subject lot
from Dalman who, in turn, purchased it from Gil. The petition was granted and a TCT was issued in Aurelio’s name.
 Remegia denied that she sold the subject lot either to Gil or Dalman. She likewise impugned as falsified the joint affidavit of confirmation of sale
that she and her uncle, Narciso Labuntog, purportedly executed before a notary public, where Remegia appears to have confirmed the sale of the
subject property to Gil. She alleged that she never parted with the certificate of title and that it was never lost. As proof that the sale of the subject
lot never transpired, Remegia pointed out that the transaction was not annotated on her TCT.
 In their answer, the spouses Zaldivar denied the material allegations in the complaint and raised the affirmative defense that Aurelio is the absolute
owner and possessor of the subject lot as evidenced by his TCT and Tax Declaration covering the same. Aurelio claimed that he acquired the
subject lot by purchase from Dalman who, in turn, bought the same from Gil on April 4, 1951. Gil allegedly purchased the subject lot from Remegia
and this sale was allegedly conformed and ratified by the latter and her uncle, Narciso Labuntog, before a notary public on December 3, 1965.
 After Aurelio obtained a loan from the GSIS, the spouses Zaldivar constructed their house on the subject lot. They alleged that they and their
predecessors-in-interest had been occupying the said property openly, publicly, adversely and continuously for over 41 years already. Aurelio filed
a petition for the issuance of a new owner’s duplicate copy of because when he asked Remegia about it, the latter claimed that it had been lost.
 The RTC rendered judgment in favor of Remegia. On appeal, the CA reversed the decision of the RTC and ruled in favor of the spouses Zaldivar.
When their MR was denied by the CA, the heirs of Feliciano (the petitioners) sought recourse to the Court in their petition for review.

ISSUE: WON the CA erred:


1. in ruling that the court who ordered the issuance of new certificate of title despite existence of owner’s duplicate copy that was never lost has
jurisdiction over the case.
2. in concluding that the respondents (defendants-appellants) are the absolute owners of the subject lot based on the TCT issued to them.
3. in concluding that petitioner’s claim of ownership over the subject lot was barred by estoppel or laches.

HELD:
WHEREFORE, the petition is GRANTED. The Decision of the CA are REVERSED and SET ASIDE. The Decision RTC of Cagayan de Oro City is
REINSTATED with the MODIFICATION that petitioners are likewise ordered to exercise the option under Article 448 of the Civil Code.

1. YES. As the trial court correctly held, the CFI which granted respondent Aurelio’s petition for the issuance of a new owner’s duplicate copy did not
acquire jurisdiction to issue such order. It has been consistently ruled that “when the owner’s duplicate certificate of title has not been lost,
but is in fact in the possession of another person, then the reconstituted certificate is void, because the court that rendered the decision had

22
no jurisdiction. Reconstitution can validly be made only in case of loss of the original certificate.” In such a case, the decision authorizing the
issuance of a new owner’s duplicate certificate of title may be attacked any time

2. YES. The court a quo correctly nullified the TCT in Aurelio’s name, emanating as it did from the new owner’s duplicate, which Aurelio procured
through fraud.

The appellate court’s reliance on the joint affidavit of confirmation of sale purportedly executed by Remegia and her uncle, Narciso Labuntog, is not
proper. In the first place, respondent Aurelio cannot rely on the joint affidavit of confirmation of sale to prove that they had validly acquired the
subject lot because, by itself, an affidavit is not a mode of acquiring ownership.Moreover, the affidavit is written entirely in English, the
hearing revealing that Feliciano does not understand English.

On this point, Article 1332 of the Civil Code is relevant:


ART.1332. When one of the parties is unable to read, or if the contract is in a language not understood by him, and mistake or fraud is alleged, the
person enforcing the contract must show that the terms thereof have been fully explained to the former.

If the person enforcing the contract fails to discharge this burden, the presumption of mistake, if not, fraud, stands unrebutted and controlling. The
bare statement of Atty. Velez (testified for the Zaldivar spouses) that he “read and interpreted” the document to the affiants and that he asked them
as to the correctness of its contents does not necessarily establish that Remegia actually comprehended or understood the import of the joint
affidavit of confirmation of sale

In a long line of cases, the Court has consistently ruled that lands covered by a title cannot be acquired by prescription or adverse possession. A
claim of acquisitive prescription is baseless when the land involved is a registered land.

Moreover, respondent Aurelio cannot raise the defense of indefeasibility of [his] title because “the principle of indefeasibility of a Torrens title does
not apply where fraud attended the issuance of the title. The Torrens title does not furnish a shield for fraud.” As such, a title issued based on void
documents may be annulled.

3. As registered owners of the lots in question, the private respondents have a right to eject any person illegally occupying their property.
This right is imprescriptible. Even if it be supposed that they were aware of the petitioner’s occupation of the property, and regardless of
the length of that possession, the lawful owners have a right to demand the return of their property at any time as long as the possession
was unauthorized or merely tolerated, if at all. This right is never barred by laches

Nonetheless, the Court is not unmindful of the fact that respondents had built their house on the subject lot and, despite knowledge thereof,
Remegia did not lift a finger to prevent it. Article 453 of the Civil Code is applicable to their case:

ART. 453. If there was bad faith, not only on the part of the person who built, planted or sowed on the land of another, but also on the
part of the owner of such land, the rights of one and the other shall be the same as though both had acted in good faith.

It is understood that there is bad faith on the part of the landowner whenever the act was done with his knowledge and without
opposition on his part.

Under the circumstances, respondents and Remegia are in mutual bad faith and, as such, would entitle the former to the application of
Article 448 of the Civil Code governing builders in good faith:

ART. 448. The owner of the land on which anything has been built, sown or planted in good faith, shall have the right to appropriate as
his own the works, sowing or planting, after payment of the indemnity provided for in Articles 54629 and 548,30 or to oblige the one who
built or planted to pay the price of the land, and the one who sowed, the proper rent. However, the builder or planter cannot be obliged
to buy the land if its value is considerably more than that of the building or trees. In such a case, he shall pay reasonable rent, if the
owner of the land does not choose to appropriate the building or trees after the proper indemnity. The parties shall agree upon the terms
of the lease and in case of disagreement, the court shall fix the terms thereof.

Following the above provision, the owner of the land on which anything has been built, sown or planted in good faith shall have the right
to appropriate as his own the building, planting or sowing, after payment to the builder, planter or sower of the necessary and useful
expenses, and in the proper case, expenses for pure luxury or mere pleasure.

Consequently, the petitioners are obliged to exercise either of the following options:
(1) to appropriate the improvements, including the house, built by the respondents on the subject lot by paying the indemnity
required by law, or
(2) sell the subject lot to the respondents.

Petitioners cannot refuse to exercise either option and compel respondents to remove their house from the land. In case petitioners choose to exercise the
second option, respondents are not obliged to purchase the subject lot if its value is considerably more than the improvements thereon and in which case,
respondents must pay rent to petitioners. If they are unable to agree on the terms of the lease, the court shall fix the terms thereof.

23
G.R. No. 176791 November 14, 2012
COMMUNITIES CAGAYAN, INC., Petitioner, vs.
SPOUSES ARSENIO (Deceased) and ANGELES NANOL AND ANYBODY CLAIMING RIGHTS UNDER THEM, Respondents.

Facts:
 Sometime in 1994, respondent-spouses Arsenio and Angeles Nanol entered into contract to Sell with petitioner Communities Cagayan, Inc.,
whereby the former agreed to sell to respondent-spouses a house and Lots 17 and 19″ locate Block 16, Camella Homes Subdivision, Cagayan
de Oro City, for the price of P 368,000.00.
 Respondent-spouses, however, did not avail of petitioner’s inhouse financing due to its high interest rates. Instead, they obtained a loan
from Capitol Development Bank, a sister company of petitioner, using the property as collateral.
 To facilitate the loan, a simulated sale over the property was executed by petitioner in favor of respondent-spouses.
 Accordingly, titles were transferred in the names of respondent-spouses under Transfer Certificates of Title (TCT) Nos. 105202 and
105203, and submitted to Capitol Development Bank for loan processing. Unfortunately, the bank collapsed and closed before it could
release the loan.
 Thus, on November 30, 1997, respondent-spouses entered into another Contract to Sell with petitioner over the same property for the same
price of P 368,000.00. This time, respondent-spouses availed of petitioner’s in-house financing thus, undertaking to pay the loan over four
years, from 1997 to 2001.
 Sometime in 2000, respondent Arsenio demolished the original house and constructed a three-story house allegedly valued at P
3.5 million, more or less.18 In July 2001, respondent Arsenio died, leaving his wife, herein respondent Angeles, to pay
for the monthly amortizations.

Issue:
Whether or not respondents are considered builders in good faith entitled to indemnification for necessary and useful expenses and/or to buy the land under
the provisions of the New Civil Code.

Held:
Yes. As a general rule, Article 448 on builders in good faith does not apply
where there is a contractual relation between the parties, such as in the instant case. We went over the records of this case and
we note that the parties failed to attach a copy of the Contract to Sell. As such, we are constrained to apply Article 448 of the
Civil Code, which provides viz:

ART. 448. The owner of the land on which anything has been built, sown or planted in good faith, shall have the right to appropriate as his own the
works, sowing or planting, after payment of the indemnity provided for in Articles 546 and 548, or to oblige the one who built or planted to pay
the price of the land, and the one who sowed, the proper rent. However, the builder or planter cannot be obliged to buy the land
if its value is considerably more than that of the building or trees. In such case, he shall pay reasonable rent, if the owner of the land
does not choose to appropriate the building or trees after proper indemnity. The parties shall agree upon the terms of the lease and in case
of disagreement, the court shall fix the terms thereof.
Article 448 of the Civil Code applies when the builder believes that he is the owner of the land or that by some title he has the right to
build thereon, or that, at least, he has a claim of title thereto. Concededly, this is not present in the instant case. The subject property is covered
by a Contract to Sell hence ownership still remains with petitioner being the seller. Nevertheless, there were already
instances where this Court applied Article 448 even if the builders do not have a claim of title over the property. Thus:

This Court has ruled that this provision covers only cases in which the builders, sowers or planters believe
themselves to be owners of the land or, at least, to have a claim of title thereto. It does not apply when the interest is merely
that of a holder, such as a mere tenant, agent or usufructuary. From these pronouncements, good faith is identified by the belief that the land
is owned; or that – by some title – one has the right to build, plant, or sow thereon.

However, in some special cases, this Court has used Article 448 by recognizing good faith beyond this limited definition. Thus,
in Del Campo v. Abesia, this provision was applied to one whose house – despite having been built at the time he was still co-
owner – overlapped with the land of another. This article was also applied to cases wherein a builder had constructed improvements with
the consent of the owner. The Court ruled that the law deemed the builder to be in good faith. In Sarmiento v. Agana, the builders were
found to be in good faith despite their reliance on the consent of another, whom they had mistakenly believed to be the owner of the land.

In fine, the Court applied Article 448 by construing good faith beyond its limited definition. We find no reason not to apply the Court’s ruling in Spouses
Macasaet v. Spouses Macasaet in this case. We thus hold that Article 448 is also applicable to the instant case. First, good faith is presumed on
the part of the respondent-spouses. Second, petitioner failed to rebut this presumption. Third, no
evidence was presented to show that petitioner opposed or objected to the improvements introduced by the respondent-
spouses. Consequently, we can validly
presume that petitioner consented to the improvements being constructed. This presumption is bolstered by the fact that as the subdivision
developer, petitioner must have given the respondent-spouses permits to commence and
undertake the construction. Under Article 453 of the Civil Code, “it is understood that there is bad faith on the part of the landowner
whenever the act was done with his knowledge and without opposition on his part.”

24
G.R. No. 201354, September 21, 2016
PABLO M. PADILLA, JR. AND MARIA LUISA P. PADILLA, Petitioners, v.
LEOPOLDO MALICSI, LITO CASINO, AND AGRIFINO GUANES, Respondents.

Facts:
 Spouses Padilla bought a parcel of land in Magsaysay Norte, Cabanatuan City in 1984.4 The lot was covered by Transfer Certificate Title No. T-
45565 and had an area of 150 square meters.5 It had an assessed value of more than P20,000.00
 Sometime in 1998, Spouses Padilla discovered that Leopoldo Malicsi, Lito Casino, and Agrifino Guanes (Malicsi, et al.) constructed houses on their
lot.
 On August 6, 2007, Spouses Padilla filed a complaint for recovery of possession against Malicsi, et al., along with three (3) others: Larry Marcelo,
Diosdado dela Cruz, and Rolando Pascua.
 In their Answer with Compulsory Counterclaim, Malicsi, et al. alleged that they believed in all honesty and good faith that the lot belonged to Toribia
Vda. De Mossessgeld (De Mossessgeld). They claimed that they possessed the land and built their houses on the lot only after receiving De
Mossessgeld's permission.
 Malicsi, et al. stated that they first found out about Spouses Padilla's claim of ownership sometime in 2002.They admitted receiving the demand
letters to vacate and pay rentals, but they refused to leave the premises.16 They denied that conciliation and mediation proceedings for amicable
settlement were ever conducted before the Katarungang Pambarangay.
 In the Decision dated July 15, 2009, the Regional Trial Court ruled that Malicsi, et al. cannot be considered as builders in good faith.
 On March 19, 2012, the Court of Appeals reversed and set aside the Regional Trial Court Decision. The Court of Appeals gave credence to Malicsi,
et al.'s allegation that they relied on De Mossessgeld's representation that she owned the lot and gave them permission to build their houses on it.
Hence, this petition.

Issue:
Whether the respondents are builders in good faith

Held:
No. A builder in good faith is a builder who was not aware of a defect or flaw in his or her title when he or she introduced improvements on a lot
that turns out to be owned by another.

Philippine National Bank v. De Jesus explains that the essence of good faith is an honest belief of the strength and validity of one's right while being
ignorant of another's superior claim at the same time:

Good faith, here understood, is an intangible and abstract quality with no technical meaning or statutory definition, and it encompasses,
among other things, an honest belief, the absence of malice and the absence of design to defraud or to seek an unconscionable
advantage. An individual's personal good faith is a concept of his own mind and, therefore, may not conclusively be determined by his
protestations alone. It implies honesty of intention, and freedom from knowledge of circumstances which ought to put the holder upon inquiry. The
essence of good faith lies in an honest belief in the validity of one's right, ignorance of a superior claim, and absence of intention to overreach
another.

Article 448 of the1 Civil Code gives a builder in good faith the right to compel the landowner to choose between two (2) options: (1) to appropriate the
building by paying the indemnity required by law; or (2) to sell the land to the builder. Ignacio v. Hilario47 summarized the respective rights of the
landowner and builder in good faith as follows:

The owner of the building erected in good faith on a land owned by another, is entitled to retain the possession of the land until he is paid the value
of his building, under article [546]. The owner of the land, upon the other hand, has the option, under article [448], either to pay for the building or to
sell his land to the owner of the building. But he cannot, as respondents here did, refuse both to pay for the building and to sell the land and compel
the owner of the building to remove it from the land where it is erected. He is entitled to such remotion only when, after having chosen to sell his
land, the other party fails to pay for the same.

Respondents say that they believed De Mossessgeld when she told them that the lot belonged to her. Yet, the records show that De Mossessgeld was a
complete stranger to them. The lack of blood relation should have been enough to put respondents on guard and convince them not to rely on her
claim of ownership. If respondents had looked into the ownership of the lot, they would have easily discovered that it was titled to petitioner Pablo
M. Padilla, Jr.'s mother as early as 1963 under Transfer Certificate of Title No. T-8303.

To support their assertion, respondents claim that they were made to believe by De Mossessgeld that she owned the lot. Respondents also claim that they
received permission from De Mossessgeld to build their houses on the land, subject to their eventual purchase of the portions where their houses stood.
However, aside from this naked and self-serving testimony, respondents failed to present any evidence to bolster their claim.

Furthermore, respondents neither presented De Mossessgeld herself nor submitted proof on which she might have based her purported ownership
of the lot. If De Mossessgeld proved elusive, respondents could then have presented statements from disinterested third parties who could testify
that it was so well-known in the community that De Mossessgeld owned the lot that they had to believe her claim of ownership. Respondents

25
likewise failed to prove that they exercised the necessary diligence required by their situation. They did not examine the tax declarations or the title to the
property before they built on it.

Failing to substantiate their claim, respondents cannot be considered as builders in good faith. Therefore, the benefits and rights provided under Article 448 of
the Civil Code do not apply. As builders in bad faith, respondents have no right to recover their expenses over the improvements they have introduced to
petitioners' lot under Article 449 of the Civil Code, which provides:

Article 449. He who builds, plants or sows in bad faith on the land of another, loses what is built, planted or sown without right to indemnity.

Under Article 452 of the Civil Code, a builder in bad faith is entitled to recoup the necessary expenses incurred for the preservation of the land.
However, respondents neither alleged nor presented evidence to show that they introduced improvements for the preservation of the land.

Therefore, petitioners as landowners became the owners of the improvements on the lot, including the residential buildings constructed by
respondents, if they chose to appropriate the accessions. However, they could instead choose the demolition of the improvements at respondents'
expense or compel respondents to pay the price of the land under Article 450 of the Civil Code

G.R. No. 182754 June 29, 2015


SPOUSES CRISPIN AQUINO and TERESA V. AQUINO, herein represented by their Attorney-in-Fact, AMADOR D. LEDESMA, Petitioners, vs.
SPOUSES EUSEBIO AGUILAR and JOSEFINA V. AGUILAR, Respondents.

Facts:
 Spouses Crispin and Teresa Aquino (petitioners) are the owners of a house and lot. Since 1981, this property has been occupied by Teresa's
sister, Josefina Aguilar and her family (respondents). Respondents stayed on the property with the consent and approval of petitioners, who were
then residing in the United States.
 While respondents were in possession of the property, the house previously constructed therein was demolished, and a three-storey building built
in its place. Respondents occupied half of the third floor of this new building for the next 20 years without payment of rental.
 Petitioners sent a letter to respondents informing them that an immediate family member needed to use the premises and demanding the surrender
of the property within 10 days from notice. Respondents failed to heed this demand, prompting petitioners to file a Complaint for ejectment against
them before the office of the barangay captain of Guadalupe Viejo. The parties attempted to reach an amicable settlement in accordance with
Section 412 of the Local Government Code, but these efforts proved unsuccessful.
 Petitioners filed a Complaint with the MeTC of Makati City praying that respondents be ordered to (a) vacate the portion of the building they were
then occupying; and (b) pay petitioners a reasonable amount for the use and enjoyment of the premises from the time the formal demand to vacate
was made.
 In their Answer with Counterclaim, respondents claimed that they had contributed to the improvement of the property and the construction of the
building, both in terms of money and management/supervision services. Petitioners purportedly agreed to let them contribute to the costs of
construction in exchange for the exclusive use of a portion of the building. Since they were allegedly co-owners of the building and builders in good
faith, respondents claimed that they had the right to be compensated for the current value of their contribution. Accordingly, they prayed for the
dismissal of the Complaint and the award of ₱5 million as compensation for their contributions to the construction of the building, as well as moral
damages, attorney's fees and costs of litigation.

Issue:
Whether respondents are builders in good faith?

Held:
No, the Spouses Aguilar cannot be considered as builders in good faith on account of their admission that the subject lot belonged to the Spouses
Aquino when they constructed the building. At the onset, petitioners were aware of a flaw in their title and a limit to their right to possess the
property. By law, one is considered in good faith if he is not aware that there exists in his title or mode of acquisition any flaw which invalidates it.

An examination of a letter sent by petitioners to Josefina Aguila abundantly shows that respondent’s occupancy of the property in question is by tolerance of
the petitioners. Said letter expressly states that the respondents are advised not to put up a shop, as the petitioners planned on disposing the property for
profit after a period of three or four years, thereby placing the respondents on notice that their possession of the said property is temporary in nature and by
mere generosity of the petitioners. The letter likewise advised them to apply for a housing project so that by the time the property in question is sold, they have
a place to transfer to.

Respondents’ contention that pursuant to Article 453 of the Civil Code, they should be considered builders in good faith even if they have acted in
bad faith, since their act of introducing improvements to one-half of the third floor of the three storey building was with knowledge and without
opposition on the part of the petitioners, cannot be sustained, principally on the ground that as stated, they were already forewarned as early not
to introduce any improvements as the property is slated to be sold as it was only bought for investment purposes. The fact that the petitioners did
not thereafter remind them of this is of no moment, as this letter was not likewise withdrawn by a subsequent one or modified by the petitioners.
That this sale did not materialize is irrelevant. What is crucial is that petitioners left respondents clear instructions not to build on the land.

In view of the foregoing, it was held that petitioners, as the owners of the land, have the right to appropriate what has been built on the property, without any
obligation to pay indemnity therefor, and that respondents have no right to a refund of any improvement built therein, pursuant to Articles 449 and 450 of the
Civil Code.

26
However, pursuant to Article 452 of the Civil Code, a builder in bad faith is entitled to recoup the necessary expenses incurred for the preservation of
the land. However, being builders in bad faith, they do not have the right of retention over the premises. While the evidence presented does not
establish the amount of necessary expenses incurred by respondents during their stay in the property, the Court noted that even petitioners do not deny that
such expenses were incurred. Accordingly, the case was remanded to the lower court for the determination of the necessary expenses of preservation of the
land, if any, incurred by respondents which expenses shall be reimbursed to them by petitioners.

By law, one is considered in good faith if he is not aware that there exists in his title or mode of acquisition any flaw which invalidates it. The presence of
evidence that petitioners prohibited the respondents from building their own structure on a portion of the property negates good faith on the part on the latter.
The fact that the petitioners did not thereafter remind respondents of this is of no moment, what is crucial is that the petitioners left the respondents clear
instructions not to build on the land.

G.R. No. 210551, June 30, 2015


JOSE J. FERRER, JR., Petitioner, v. CITY MAYOR HERBERT BAUTISTA, CITY COUNCIL OF QUEZON CITY, CITY TREASURER OF QUEZON CITY,
AND CITY ASSESSOR OF QUEZON CITY, Respondents.

Facts:
 Quezon City Council enacted Ordinance No. SP-2095, S-2011, or the Socialized Housing Tax of Quezon City, Section 3 of which provides:

SECTION 3. IMPOSITION. A special assessment equivalent to one-half percent (0.5%) on the assessed value of land in excess of One Hundred
Thousand Pesos (Php100,000.00) shall be collected by the City Treasurer which shall accrue to the Socialized Housing Programs of the Quezon
City Government.

“Effective for five (5) years, the Socialized Housing Tax (SHT) shall be utilized by the Quezon City Government for the following projects: (a) land
purchase/land banking; (b) improvement of current/existing socialized housing facilities; (c) land development; (d) construction of core houses,
sanitary cores, medium-rise buildings and other similar structures; and (e) financing of public-private partnership agreement of the Quezon City
Government and National Housing Authority (NHA) with the private sector. “

 On the other hand, Ordinance No. SP-2235, S-2013 was enacted on December 16, 2013 and took effect ten days after when it was approved by
respondent City Mayor.6 The proceeds collected from the garbage fees on residential properties shall be deposited solely and exclusively in an
earmarked special account under the general fund to be utilized for garbage collections.7 Section 1 of the Ordinance set forth the schedule and
manner for the collection of garbage fees:

The collection of the garbage fee shall accrue on the first day of January and shall be paid simultaneously with the payment of the real property tax,
but not later than the first quarter installment.8 In case a household owner refuses to pay, a penalty of 25% of the garbage fee due, plus an interest
of 2% per month or a fraction thereof, shall be charged. ChanRoblesVi

 Petitioner Ferrer claims that the annual property tax is an ad valorem tax, a percentage of the assessed value of the property, which is subject to
revision every three (3) years in order to reflect an increase in the market value of the property. The SHT and the garbage fee are actually
increases in the property tax which are not based on the assessed value of the property or its reassessment every three years; hence, in violation
of Sections 232 and 233 of the LGC.

Issue:
Whether or not the tax ordinances are valid

Held:
Ordinance No. SP-2095, S-2011, the Socialized Housing Tax is valid. Ordinance No. SP-2235, S-2013, which collects an annual garbage fee on all domestic
households in Quezon City, is hereby declared as UNCONSTITUTIONAL AND ILLEGAL

The 1987 Constitution explicitly espouses the view that the use of property bears a social function and that all economic agents shall contribute to the common
good. Property has not only an individual function, insofar as it has to provide for the needs of the owner, but also a social function insofar as it has to provide
for the needs of the other members of society. The principle is this:
Police power proceeds from the principle that every holder of property, however absolute and unqualified may be his title, holds it under the implied liability
that his use of it shall not be injurious to the equal enjoyment of others having an equal right to the enjoyment of their property, no r injurious to the right of the
community.

Property rights of individuals may be subjected to restraints and burdens in order to fulfill the objectives of the government in the exercise of police power. In
this jurisdiction, it is well-entrenched that taxation may be made the implement of the state’s police power.

The SHT charged by the Quezon City Government is a tax which is within its power to impose. Cities are allowed to exercise such other powers and discharge
such other functions and responsibilities as are necessary, appropriate, or incidental to efficient and effective provision of the basic services and facilities
which include, among others, programs and projects for low-cost housing and other mass dwellings. The collections made accrue to its socialized housing
programs and projects.

27
The tax is not a pure exercise of taxing power or merely to raise revenue; it is levied with a regulatory purpose. The levy is primarily in the exercise of the
police power for the general welfare of the entire city. It is greatly imbued with public interest. Removing slum areas in Quezon City is not only beneficial to the
underprivileged and homeless constituents but advantageous to the real property owners as well. The situation will improve the value of the their property
investments, fully enjoying the same in view of an orderly, secure, and safe community, and will enhance the quality of life of the poor, making them law-
abiding constituents and better consumers of business products.

In the subject ordinance imposing garbage collection fee, the rates of the imposable fee depend on land or floor area and whether the payee is an occupant of
a lot, condominium, social housing project or apartment. For easy reference, the relevant provision is again quoted below:

The rates being charged by the ordinance are unjust and inequitable: a resident of a 200 sq. m. unit in a condominium or socialized housing project
has to pay twice the amount than a resident of a lot similar in size; unlike unit occupants, all occupants of a lot with an area of 200 sq. m. and less
have to pay a fixed rate of Php100.00; and the same amount of garbage fee is imposed regardless of whether the resident is from a condominium
or from a socialized housing project.

Indeed, the classifications under Ordinance No. S-2235 are not germane to its declared purpose of "promoting shared responsibility with the
residents to attack their common mindless attitude in over-consuming the present resources and in generating waste." Instead of simplistically
categorizing the payee into land or floor occupant of a lot or unit of a condominium, socialized housing project or apartment, respondent City
Council should have considered factors that could truly measure the amount of wastes generated and the appropriate fee for its collection. Factors
include, among others, household age and size, accessibility to waste collection, population density of the barangay or district, capacity to pay,
and actual occupancy of the property. R.A. No. 9003 may also be looked into for guidance. Under said law, WM service fees may be computed based on
minimum factors such as types of solid waste to include special waste, amount/volume of waste, distance of the transfer station to the waste management
facility, capacity or type of LGU constituency, cost of construction, cost of management, and type of technology. With respect to utility rates set by
municipalities, a municipality has the right to classify consumers under reasonable classifications based upon factors such as the cost of service,
the purpose for which the service or the product is received, the quantity or the amount received, the different character of the service furnished,
the time of its use or any other matter which presents a substantial difference as a ground of distinction.

G.R. No. L-61647 October 12, 1984


REPUBLIC OF THE PHILIPPINES (DIRECTOR OF LANDS), petitioner,
vs.
THE HON. COURT OF APPEALS, BENJAMIN TANCINCO, AZUCENA TANCINCO REYES, MARINA TANCINCO IMPERIAL and MARIO C.
TANCINCO, respondents.

FACTS:
 Private Respondents are registered owners of a parcel of land situated at Barrio Ubihan, Meycauyan, Bulacan bordering on Meycauyan and
Bocaue Rivers. They filed an application for the registration of three lots adjacent to their fishpond property which was opposed by the Bureau of
Lands. The trial proceeded only with respect to Lots 1 and 2, as Private Respondents withdrew their application to Lot 3.
 The lower court rendered its decision granting the application on the finding that the lands are accretions to Private Respondents’ fishponds which
was later on affirmed by the CA.
 The Petitioner argued that there’s no accretion because what actually happened was that the respondents transferred their dikes further down the
river bed of the Meycauyan River, and thus, if there is any accretion, it is man-made and artificial and not the result of the gradual and imperceptible
sedimentation by the waters of the river. The Private Respondents denied the allegations and alleged that there was no human intervention
because the transfer of dike occurred after the accretion was complete.

ISSUE:
Whether or not the subject lands are accretions to the Private Respondents’ fishponds.

RULING:
No.
Article 457 of the New Civil Code which provides that: “To the owners of lands adjoining the banks of rivers belong the accretion which they
gradually receive from the effects of the current of the waters.” requires the concurrence of three requisites before an accretion covered by this particular
provision is said to have taken place. They are (1) that the deposit be gradual and imperceptible; (2) that it be made through the effects of the current
of the water; and (3) that the land where accretion takes place is adjacent to the banks of rivers. The requirement that the deposit should be due to the
effect of the current of the river is indispensable. This excludes from Art. 457 of the New Civil Code all deposits caused by human intervention.
Alluvion must be the exclusive work of nature.
In the instant case, there is no evidence whatsoever to prove that the addition to the said property was made gradually through the effects of the
current of the Meycauayan and Bocaue rivers. The Private Respondents rely only on the testimony of Mrs. Virginia Acuna. However, there is evidence that the
alleged alluvial deposits were artificial and man-made and not the exclusive result of the current of the Meycauayan and Bocaue rivers. The alleged alluvial
deposits came into being not because of the sole effect of the current of the rivers but as a result of the transfer of the dike towards the river and encroaching
un it.
Hence, the lots were portions of the bed of the Meycauayan river and are therefore classified as property of the public domain under Article 420
paragraph 1 and Article 502, paragraph 1 of the Civil Code of the Philippines.

G.R. No. 95907 April 8, 1992

28
JOSE REYNANTE, petitioner,
vs.
THE HONORABLE COURT OF APPEALS, THE HON. VALENTIN CRUZ, as Presiding Judge, Regional Trial Court of Bulacan, Branch VIII, and the
HEIRS OF LEONCIO CARLOS and DOLORES A. CARLOS, and HEIRS OF GORGONIO CARLOS and CONCEPCION CARLOS, respondents.

FACTS:
 The Petitioner was taken as tenant by Don Cosme Carlos, owner and father-in-law of the Private Respodents, over a fishpond located at Barrio
Liputan, Meycauyan, Bulacan where the former constructed a nipa hut and planted nipa palms on lots 1 and 2. Petitioner harvested and sold said
nipa palms without interference and prohibition from anybody, as well as Don Cosme Carlos who did not question his right to plant the nipa palms
near the fishpond.
 After the death of Don Cosme Carlos, his heirs, the Private Respondents, entered into a written agreement with the petitioner whereby the latter
turned over the fishpond to them in consideration of money. Private Respondents then leased the said fishpond to Carlos Dela Cruz. However, the
Petitioner continued to live in the nipa hut and took care the nipa palms he had planted.
 The Private Respondents formally demanded the petitioner to vacate said portion of land which the latter refused. The former then filed a complaint
for forcilble entry against the petitioner which was dismissed by the trial court. On respondent’s appeal, the RTC reversed the decision of the lower
court which was later on affirmed by the CA.

ISSUE:
Whether or not the disputed lots belong to private respondents as a result of accretion.

RULING:
No.
As the Supreme Court held in the case of Ignacio Grande, et al. vs. CA, et. Al., GR No. L-17652, an accretion does not automatically become
registered land just because the lot which receives such accretion is covered by a Torrens Title. Ownership of a piece of land is one thing;
registration under the Torrens system of that ownership is another. Ownership over the accretion received by the land adjoining a river is governed by
the Civil Code. Imprescriptibility of registered land is provided in the registration law. Registration under the Land Registration and Cadastral Act does not
vest or give title to the land, but merely confirms and, thereafter, protects the title already possessed by the owner, making it imprescriptible by
occupation of third parties. But to obtain this protection, the land must be placed under the operation of the registration laws, wherein certain
judicial procedures have been provided.
In the present case, assuming private respondents had acquired the alluvial deposit (the lot in question), by accretion, still their failure to register
said accretion for a period of fifty (50) years subjected said accretion to acquisition through prescription by third persons. Moreover, it is undisputed that
petitioner has been in possession of the subject lots for more than fifty (50) years and unless private respondents can show a better title over the subject lots,
petitioner's possession over the property must be respected.
Hence, the Petitioner has better rights over the disputed lots through prescription.

G.R. No. 178411 June 23, 2010


OFFICE OF THE CITY MAYOR OF PARAÑAQUE CITY, OFFICE OF THE CITY ADMINISTRATOR OF PARAÑAQUE CITY, OFFICE OF THE CITY
ENGINEER OF PARAÑAQUE CITY, OFFICE OF THE CITY PLANNING AND DEVELOPMENT COORDINATOR, OFFICE OF THE BARANGAY CAPTAIN
AND SANGGUNIANG PAMBARANGAY OF BARANGAY VITALEZ, PARAÑAQUE CITY, TERESITA A. GATCHALIAN, ENRICO R. ESGUERRA,
ERNESTO T. PRACALE, JR., MANUEL M. ARGOTE, CONRADO M. CANLAS, JOSEPHINE S. DAUIGOY, ALLAN L. GONZALES, ESTER C. ASEHAN,
MANUEL A. FUENTES, and MYRNA P. ROSALES, Petitioners,
vs.
MARIO D. EBIO AND HIS CHILDREN/HEIRS namely, ARTURO V. EBIO, EDUARDO V. EBIO, RENATO V. EBIO, LOURDES E. MAGTANGOB, MILA V.
EBIO, and ARNEL V. EBIO, Respondents.

FACTS:
 Respondents claim they are the absolute owners of a parcel of lands in Barangay Vitalez, Paranaque City under the name of respondent Mario
Ebio. Said land was an accretion of Cut-cut creek. Respondents asserted that the original occupant and possessor of said land was their great
grandfather, Jose Vitalez, who later on transferred to his son, Pedro Vitalez, which the latter constinously and exclusively occupied the said lot.
Since then, respondents have been paying real property taxes for the said property.
 Mario Ebio married Pedro’s daughter, Zenaida, where they established their home on the said lot. Thereafter, Pedro executed a notarized Transfer
of Rights ceding his claim over the entire parcel of land to Mario Ebio. However, the Sangguiang Barangay of Vitalez passed a resolution seeking
assistance from City of Paranaque for the construction of an access road along Cut-cut creek located in the said barangay. The proposed road will
traverse the lot occupied by the respondents which was latter later on opposed. However, respondents were surprised when several officials from
the barangay and the city planning office proceeded to cut eight coconut trees planted on the said lot.
 The City Administrator sent a letter to the respondents ordering them to vacate the area within the next thirty (30) days, or be physically evicted
from the said property. The latter asserted their claim over the subject property but such request remained unheeded.
 The respondent went to RTC of Paranaque and applied for a writ of preliminary injunction against petitioners which was later on denied stating that
private respondents were not able to prove successfully that they have an established right to the property since they have not instituted an action
for confirmation of the title and their application for sales patent has not been granted. On respondent’s appeal, the CA reversed the decision of the
lower court stating that, based on evidentiary records, Pedro Vitalez possessed the accreted property since 1930 and said property became subject
to tax which were paid by the Respondents.

29
ISSUE:
Whether or not the Private Respondents has better right over the subject property.

RULING:
Yes.
Under Article 457, alluvial deposits along the banks of a creek do not form part of the public domain as the alluvial property automatically
belongs to the owner of the estate to which it may have been added. The only restriction provided for by law is that the owner of the adjoining
property must register the same under the Torrens system; otherwise, the alluvial property may be subject to acquisition through prescription by
third persons.
In the instant case, for more than thirty (30) years, neither Guaranteed Homes, Inc. nor the local government of Parañaque in its corporate or
private capacity sought to register the accreted portion. Undoubtedly, respondents are deemed to have acquired ownership over the subject property through
prescription. Respondents can assert such right despite the fact that they have yet to register their title over the said lot. It is clear that since 1930,
Respondents together with their predecessor-in-interest, PEDRO VITALEZ, have been in exclusive possession of the subject property and starting 1964 had
introduced improvements thereon as evidenced by their construction permits. Thus, even by extraordinary acquisitive prescription, Respondents have
acquired ownership of the property in question since 1930 even if the subject land was subsequently registered in the name of Guaranteed Homes.

G.R. No. 82220 July 14, 1995


PABLITO MENESES and LORENZO MENESES, petitioners,
vs.
THE HONORABLE COURT OF APPEALS, EDUARDO QUISUMBING, NORBERTO QUISUMBING, HEIRS OF EMILIO QUISUMBING (Carlos, Manuel
and Paz, all surnamed Quisumbing), HEIRS OF FERNANDO QUISUMBING (Perla, Josefina, Napoleon, Honorato, Remedios and Alfonso, all
surnamed Quisumbing), HEIRS OF MANUEL QUISUMBING, SR. (Petrona, Natividad, Manuel, Jr., Dolores and Lilia, all surnamed Quisumbing) and
HEIRS OF FRANCISCO QUISUMBING (Fe, Johnny, Ma. Luisa, Norberto, Jimmy, Ma. Victoria, Elsa and Oscar, all surnamed Quisumbing), all
represented by Atty. Galileo Brion, respondents.

G.R. No. 82251 July 14, 1995


CESAR ALMENDRAL, petitioner,
vs.
EDUARDO QUISUMBING, respondent.

G.R. No. 83059 July 14, 1995


EDUARDO QUISUMBING, NORBERTO QUISUMBING, HEIRS OF EMILIO QUISUMBING (Carlos, Manuel and Paz, all surnamed Quisumbing), HEIRS
OF FERNANDO QUISUMBING, (Perla, Josefina, Napoleon, Honorato, Remedios and Alfonso, all surnamed Quisumbing), HEIRS OF MANUEL
QUISUMBING, SR. (Petrona, Natividad, Manuel, Jr., Dolores and Lilia, all surnamed Quisumbing) and HEIRS OF FRANCISCO QUISUMBING (Fe,
Johnny, Ma. Victoria, Elsa and Oscar, all surnamed Quisumbing), petitioners,
vs.HON. COURT OF APPEALS, PABLITO MENESES, LORENZO MENESES and BRAULIO C. DARUM, respondents.

FACTS:
 On March 1, 1977, Braulio C. Darum, then the District Land Officer, issued to Petitioner Pablito Meneses free patent and titles of Lot 1585 and Lot
190 located in Los Banos, Laguna. The latter acquired said lots from Silverio Bautista through Deed of Waiver and Transfer of Rights in
consideration of “love and affection” and “some monetary obligations” which he later on took possession. On the other hand, the Quisumbing family
traces ownership of the same land as far back as September 6, 1919 when their matriarch, Ciriaca Quisumbing, was issued Original Certificate
Title which was then registered in the name of her heirs, the Petitioners.
 The Quisumbing instituted an accion publiciana in the CFI of Binan, Laguna to recover possession over the property from Dominga Villamor and
Lorenzo Lanuzo which was later on decided on their favor, as well as the appeal in the CA.
 The Quisumbing then applied for registration and confirmation of title over an additional land area which had gradually accrued to their property by
the natural action of the waters of Laguna de Bay which was later on confirmed by CFI of Binan, Laguna.
 The Quisumbing then filed before the CFI of Calamba, Laguna for the nullification of the free patents and titles issued to Pablito Meneses alleging
that Lorenzo Meneses, then Mayor of Los Banos, Laguna, used his brother Pablito as a “tool and dummy,” illegally occupied their “private accretion
land.” The trial court rendered the decision finding that the lands registered by the Meneses brothers are accretion lands to which the Quisumbings
have a valid right and found that the free patents issued to Pablito Meneses had been procured through fraud, deceit and bad faith. Thereafter, the
Quisumbings filed a motion for execution pending appeal which the trial court granted. Meanwhile, the CA affirmed the lower court’s decision.

ISSUE:
Whether or not the lands in question were not accretion lands but lands of the public domain.

RULING:
No.
As held in Republic vs. CA, Laguna de Bay is a lake. While the waters of a lake are also subject to the same gravitational forces that cause
the formation of tides in seas and oceans, this phenomenon is not a regular daily occurrence in the case of lakes. Thus, the alternation of high
tides and low tides, which is an ordinary occurrence, could hardly account for the rise in the water level of the Laguna de Bay as observed four to
five months a year during the rainy season. Rather, it is the rains which bring about the inundation of a portion of the land in question. Since the
rise in the water level which causes the submersion of the land occurs during a shorter period (four to five months a year) than the level of the

30
water at which the land is completely dry, the latter should be considered as the "highest ordinary depth" of Laguna de Bay. Therefore, the land
sought to be registered is not part of the bed or basin of Laguna de Bay. Neither can it be considered as foreshore land.
Moreover, Accretion as a mode of acquiring property under Article 457 of the Civil Code requires the concurrence of these requisites: (1)
that the deposition of soil or sediment be gradual and imperceptible; (2) that it be the result of the action of the waters of the river (or sea); and (3)
that the land where accretion takes place is adjacent to the banks of rivers (or the sea coast).
In the case at bar, while the trial court has not determined on its own the presence of said requisites, it is too late now for petitioners Pablo and
Lorenzo Meneses to claim otherwise. Consequently, the lands held to be accretion lands could only benefit the Quisumbings, who own the property adjacent
to the lands in controversy.

G.R. No. 160453 November 12, 2012


REPUBLIC OF THE PHILIPPINES, Petitioner,
vs.
ARCADIO IVAN A. SANTOS III, and ARCADIO C. SANTOS, JR., Respondents.

FACTS:
 Alleging continuous and adverse possession for more than 10 years, Respondent Arcadio Ivan Santos III applied for the registration before RTC
Paranaque of Lot 4998-B located in Barangay San Dionisio, Paranaque City alleging that the said property had been formed through accretion and
had been in their joint, open, notorious, public, continuous and adverse possession for more than 30 years. The application was opposed by the
City of Paranaque contending that the title of the property could not be registered in favor of the respondents for the reason that the property was
an orchard that had dried up and had not resulted from accretion.
 The RTC granted the application for land registration of the Respondents which was later on affirmed by the CA.

ISSUE:
Whether or not the drying up of the river is an accretion.

RULING:
No.
Under Article 457, Accretion is the process whereby the soil is deposited along the banks of rivers. The deposit of soil, to be considered
accretion, must be: (a) gradual and imperceptible; (b) made through the effects of the current of the water; and (c) taking place on land adjacent to
the banks of rivers.
In the present case, the land was definitely not an accretion. The process of drying up of a river to form dry land involved the recession of
the water level from the river banks, and the dried-up land did not equate to accretion, which was the gradual and imperceptible deposition of soil
on the river banks through the effects of the current. In accretion, the water level did not recede and was more or less maintained. Hence,
respondents as the riparian owners had no legal right to claim ownership of Lot 4998-B. Considering that the clear and categorical language of Article 457 of
the Civil Code has confined the provision only to accretion, we should apply the provision as its clear and categorical language tells us to. Axiomatic it is,
indeed, that where the language of the law is clear and categorical, there is no room for interpretation; there is only room for application.16 The first and
fundamental duty of courts is then to apply the law.
Therefore, the State exclusively owned Lot 4998-B and may not be divested of its right of ownership. Article 502 of the Civil Code expressly
declares that rivers and their natural beds are public dominion of the State.18 It follows that the river beds that dry up, like Lot 4998-B, continue to belong to the
State as its property of public dominion, unless there is an express law that provides that the dried-up river beds should belong to some other person

CUREG VS. IAC

G.R. No. 73465 September 7, 1989

LEONIDA CUREG, ROMEO, PEPITO, HERNANDO, MANUEL, ANTONIO AND ELPIDIO (ALL SURNAMED CARNIYAN) petitioner,
vs.
INTERMEDIATE APPELLATE COURT, (4TH CIVIL CASES DIVISION), DOMINGO APOSTOL, SOLEDAD GERARDO, ROSA GERARDO, NIEVES
GERARDO, FLORDELIZA GERARDO, AND LILIA MAQUINAD, respondent.

FACTS:

On 5 November 1982, Domingo Apostol, Soledad Gerardo, Rosa Gerardo, Nieves Gerardo, Flordeliza Gerardo and Lilia Maquinad filed a complaint for
quieting of title and damages with preliminary injunction against Leonida, Romeo, Pepito, Hernando, Manuel, Antonio and Elpidio Carniyan with the RTC
Isabela

A temporary restraining order was issued by the trial court on 12 November 1982.

The complaint alleged that the Gerardos and Maquinad are the legal and/or the forced heirs of the late Domingo Gerardo, who died in February 1944, the
latter being the only issue of the late Francisco Gerardo, who died before the outbreak of WWII; that since time immemorial and/or before 26 July 1894, the

31
late Francisco Gerardo, together with his predecessors-in-interest have been in actual, open, peaceful and continuous possession, under a bona fide claim of
ownership and adverse to all other claimants, of a parcel of land, situated in Casibarag-Cajel, Cabagan, Isabela, containing an area of 2.5 hectares

Said land was declared for taxation purposes in the name of Francisco Gerardo; that upon the death of Francisco Gerardo, the ownership and possession of
the land was succeeded by his only issue, Domingo Gerardo who, together with 3 legal or forced heirs, namely Soledad Gerardo, Primo Gerardo(+) and Salud
Gerardo(+) have also been in actual, open, peaceful and continuous possession of the same.

Primo Gerardo was survived by Rosa, Nieves and Flordeliza Gerardo; while Salud Gerardo was survived by Lilia Maquinad. In 1979, Soledad, Rosa, Nieves,
and Flordeliza Gerardo along with Lilia Maquinad verbally sold the land to Domingo Apostol. On 10 September 1982, the verbal sale and conveyance was
reduced into writing by the vendors who executed an "Extra-Judicial Partition with Voluntary Reconveyance.”

About the time of the execution of the Extra-Judicial Partition, the land already manifested signs of accretion of about 3 hectares on the north caused by the
northward movement of the Cagayan River; that Domingo Apostol declared the land and its accretion for tax purposes. Sometime about the last week of
September and or the first week of October 1982, when the Gerardos, Maquinad and Apostol were about to cultivate their land together with its accretion, they
were prevented and threatened by the Carniyans (Leonida Cureg and Romeo, Pepito, Hernando, Manuel, Antonio and Elpidio: surviving spouse and children
of Antonio Carniyan) from continuing to do so.

The late Antonio Carniyan was the owner of a piece of land situated in Casibarag-Cajel, Cabagan, Isabela which contained an area of 2,790 sq. m. and which
was declared for taxation purposes. Carniyan revised on 28 November 1968 his TD 13131 dated 24 July 1961 to conform with the correct area and
boundaries of his OCT P-19093 issued on 25 November 1968 pursuant to Free Patent 399431 dated 21 May 1968; that the area under the new TD 15663
was increased from 2,790 sq.ms. to 4,584 sq.ms. and the boundary on the north became Cagayan River, purposely eliminating completely the original
boundary on the north which is Domingo Gerardo. The heirs of Antonio Carniyan (Cureg, et.al.) alleged in their answer that the land claimed by the Gerardos
and Apostol is non-existent; that Antonio Carniyan was the owner of a piece of land bounded on the north by Cagayan River and not by the land of Francisco
Gerardo; that the "subject land" is an accretion to their registered land and that they have been in possession and cultivation of the "accretion" for many years.

The application for the issuance of a writ of preliminary injunction was denied on 28 July 1983 on the ground that the Carniyans (Cureg) were in actual
possession of the land in litigation prior to September 1982. In a decision rendered on 6 July 1984, the trial court rendered judgment declaring Domingo
Apostol the absolute owner of the parcel of land containing an area of 5.5000 hectares and with an assessed value of P3,520; ordering the issuance of a writ
of preliminary injunction against Cureg, et.al.; ordering that the writ be made permanent; and ordering Cureg, et.al. to pay Apostol, et.al. a reasonable
attorney's fee of P5,000.00, litigation expenses of P1,500.00 and costs.

On 17 July 1984, Cureg appealed to the then IAC Court which affirmed the decision of the trial court. Cureg's Motion for Reconsideration was denied on 8
January 1986. Hence, the petition for review under Rule 45 of the Rule of Court.

RULING:

1. Tax Declaration not sufficient evidence to prove ownership; OCT indicates true and legal ownership
Gerardos' and Maquinad’s (therefore Apostol’s) claim of ownership of their alleged 2.5 hectares of land is anchored mainly on 4 tax declarations. The
declaration of ownership for purposes of assessment on the payment of the tax is not sufficient evidence to prove ownership. (Evangelista v. Tabayuyong, 7
Phil. 607; Elumbaring v. Elumbaring, 12 Phil. 384; cited in Camo v. Riosa Bayco, 29 Phil. 437, 444). On the other hand, Cureg et.al. relied on the
indefeasibility and incontrovertibility of their OCT P-19093. In the case of Ferrer-Lopez v. Court of Appeals (GR 50420, 29 May 1987, 150 SCRA 393, 401-
402), it was ruled that as against an array of proofs consisting of tax declarations and/or tax receipts which are not conclusive evidence of ownership nor proof
of the area covered therein, an original certificate of title indicates true and legal ownership by the registered owners over the disputed premises. Cureg's OCT
P-19093 should be accorded greater weight as against the tax declarations offered by Apostol, et.al. in support of their claim, which declarations are all in the
name of the latters’ predecessor-in-interest, Francisco Gerardo, and appear to have been subscribed by him after the last war, when it was established during
the trial that Francisco Gerardo died long before the outbreak of the last war.

2. Decree of registration bars all claims and rights arising or existing prior to decree
A decree of registration bars all claims and rights which arose or may have existed prior to the decree of registration. By the issuance of the decree, the land is
bound and title thereto quieted, subject only to exceptions stated in Section 39, Act 496 (now Section 44 of PD 1529). Since Cureg's original certificate of title
clearly stated that subject land is bounded on the north by the Cagayan River, Apostol's claim over the land allegedly existing between Cureg's land and the
Cagayan River, is deemed barred and nullified with the issuance of the original certificate of title.

3. Tax Declaration of earlier date cannot defeat OCT of later date; Cureg not estopped as Tax Declarations subsequent to issuance of OCT states
northern boundary is Cagayan river
A tax declaration, being of an earlier date cannot defeat an original certificate of title which is of a later date. The appellate court erred in considering Tax
Declaration 13131, in the name of Antonio Carniyan, as an admission by him that his land is bounded on the north by the land of Domingo Gerardo and thus is
estopped from claiming otherwise. The tax declarations of the late Antonio Carniyan subsequent to the issuance of OCT P-19093 already states that its
northern boundary is Cagayan River. In effect, he has repudiated any previous acknowledgment by him, granting that he caused the accomplishment of the
tax declarations in his name before the issuance of OCT P-19093, of the existence of Francisco Gerardo's land.

32
4. Cureg, et.al. in actual possession; Evidence

Evidence on record proves that Cureg, et.al. are in actual possession of the land. First, the trial court in its Decision stated the reason for denying
private respondents' petition for the issuance of a preliminary injunction, is that Cureg, et.al. were in actual possession of the land in litigation prior
to September 1982. Second, witness for Apostol, et.al., Esteban Guingab, boundary owner on the east of the land in question and whose own land
is bounded on the north of Cagayan River, on cross-examination, revealed that when his property was only more than 1 hectare in 1958, (now more
than 4 hectares) his boundary on the west is the land of Antonio Carniyan. Third, witness Rogelio C. Albano, a geodetic engineer, on direct
examination stated that in 1974, the late Antonio Carniyan requested him to survey the land covered by his title and the accretion attached to it, but
he did not pursue the same because he learned from the Office of the Director of the Bureau of Lands that the same accretion is the subject of an
application for homestead patent of one Democrata Aguila, contrary to the statement of the trial court and the appellate court that Albano "made
three attempts to survey the land but he did not continue to survey because persons other than defendants were in possession of the land," which
statement appears only to be a conclusion. Fourth, an order by the Director of Lands dated 14 August 1980 in connection with the Homestead
Application of Democrata Aguila of an accretion situated in Catabayungan, Cabagan, Isabela, such application was disapproved because in an
investigation conducted by the Bureau of Lands of the area applied for which is an accretion, the same was found to be occupied and cultivated
by, among others, Antonio Carniyan, who claimed it as an accretion to his land. Apostol, et.al. nor their predecessors-in-interest appeared as one
of those found occupying and cultivating said accretion.

5. Accretion belongs to riparian owners

The land in question is an alluvial deposit left by the northward movement of the Cagayan River and pursuant to Article 457 of the New Civil Code,
it is said that "to the owners of land adjoining the banks of river belong the accretion which they gradually receive from the effects of the current of
the waters."

6. Accretion does not automatically become registered land

The area covered by OCT P-19093 is only 4,584 sq. ms. The accretion attached to said land is approximately 5.5 hectares. The increase in the area
of Cureg's land, being an accretion left by the change of course or the northward movement of the Cagayan River does not automatically become
registered land just because the lot which receives such accretion is covered by a Torrens title. As such, it must also be placed under the
operation of the Torrens System.

JAGUALING VS. CA

G.R. No. 94283 March 4, 1991

MAXIMO JAGUALING, ANUNCITA JAGUALING and MISAMIS ORIENTAL CONCRETE PRODUCTS, INC.,petitioners,
vs.
COURT OF APPEALS (FIFTEENTH DIVISION), JANITA F. EDUAVE and RUDYGONDO EDUAVE, respondents.

Facts:

A certain parcel of land is located in Sta. Cruz, Tagoloan, Misamis Oriental with an area of 16,452 sq. m., forming part of an island in a non-navigable river,
bounded by the Tagoloan River on the north, south, and east and by the portion belonging to Vicente Neri on the west. Janita Eduave claims that she inherited
the land from her father, Felomino Factura, together with his co-heirs, Reneiro Factura and Aldenora Factura, and acquired sole ownership of the property by
virtue of a Deed of Extra Judicial Partition with sale.

The land is declared for tax purposes under Tax Declaration 26137 with an area of 16,452 sq. m. Since the death of her father on 5 May 1949, Eduave had
been in possession of the property although the tax declaration remains in the name of the deceased father. The entire land had an area of 16,452 sq. m.
appearing in the deed of extrajudicial partition, while in tax declaration the area is only 4,937 sq. m., and she reasoned out that she included the land that was
under water. The land was eroded sometime in November 1964 due to typhoon Ineng, destroying the bigger portion and the improvements leaving only a
coconut tree. In 1966 due to the movement of the river deposits on the land that was not eroded increased the area to almost half a hectare and in 1970
Eduave started to plant banana trees.

In 1973, Maximo and Anuncita Jagualing asked her permission to plant corn and bananas provided that they prevent squatters to come to the area. Eduave
engaged the services of a surveyor who conducted a survey and placed concrete monuments over the land. Eduave also paid taxes on the land in litigation,
and mortgaged the land to the Luzon Surety and Co., for a consideration of P6,000.00. The land was the subject of a reconveyance case between Janita
Eduave vs. Heirs of Antonio Factura, which was the subject of judgment by compromise in view of the amicable settlement of the parties. The heirs of Antonio
Factura had ceded a portion of the land with an area of 1,289 sq. m., to Janita Eduave in a notarial document of conveyance, pursuant to the decision of the
CFI, after a subdivision of the lot 62 Pls-799, and containing 1,289 sq. m. was designated as Lot 62-A, and the subdivision plan was approved. Eduave also
applied for concession with the Bureau of Mines to extract 200 m3 of grave, and after an ocular inspection the permit was granted. Eduave, after permit was
granted, entered into an agreement with Tagoloan Aggregates to extract sand and gravel, which agreement was registered in the office of the Register of
Deeds. Maximo and Anuncita Jagualing assert that they are the real owners of the land in litigation containing an area of 18,000 sq. m. During the typhoon

33
Ineng in 1964 the river control was washed away causing the formation of an island. Jagualing started occupying the land in 1969, paid land taxes as
evidenced by tax declaration 26380 and tax receipts, and tax clearances. Actual occupation of the land by Jagualing included improvements and the house.

Rudygondo and Janita Eduave filed with the RTC Misamis Oriental an action to quiet title and/or remove a cloud over the property in question against
Jagualing. On 17 July 1987 the trial court dismissed the complaint for failure of Eduave to establish by preponderance of evidence their claim of ownership
over the land in litigation. The court found that the island is a delta forming part of the river bed which the government may use to reroute, redirect or control
the course of the Tagoloan River. Accordingly, it held that it was outside the commerce of man and part of the public domain, citing Article 420 of the Civil
Code. As such it cannot be registered under the land registration law or be acquired by prescription. The trial court, however, recognized the validity of
Jagualing’s possession and gave them preferential rights to use and enjoy the property. The trial court added that should the State allow the island to be the
subject of private ownership, the Jagualings have rights better than that of Eduave.

On appeal to the CA, the court found that the island was formed by the branching off of the Tagoloan River and subsequent thereto the accumulation of
alluvial deposits. Basing its ruling on Articles 463 and 465 of the Civil Code, the CA reversed the decision of the trial court, declared Eduave as the lawful and
true owners of the land subject of the case and ordered Jagualing to vacate the premises and deliver possession of the land to Eduave.

Issue:

Who between the one who has actual possession of an island that forms in a non-navigable and non-floatable river and the owner of the land along the margin
nearest the island, has the better right thereto?

Held:

The parcel of land is part of an island that formed in a non-navigable and non-floatable river; from a small mass of eroded or segregated outcrop of land, it
increased to its present size due to the gradual and successive accumulation of alluvial deposits. The CA did not err in applying Article 465 of the Civil Code.
Under this provision, the island belongs to the owner of the land along the nearer margin as sole owner thereof; or more accurately, because the island is
longer than the property of Eduave, they are deemed ipso jure to be the owners of that portion which corresponds to the length of their property along the
margin of the river.

Lands formed by accretion belong to the riparian owner. This preferential right is, under Article 465, also granted the owners of the land located in the margin
nearest the formed island for the reason that they are in the best position to cultivate and attend to the exploitation of the same. In fact, no specific act of
possession over the accretion is required. If, however, the riparian owner fails to assert his claim thereof, the same may yield to the adverse possession of
third parties, as indeed even accretion to land titled under the Torrens system must itself still be registered.

There is no need to make a final determination regarding the origins of the island, i.e., whether the island was initially formed by the branching off or division of
the river and covered by Article 463 of the Civil Code, in which case there is strictly no accession because the original owner retains ownership, or whether it
was due to the action of the river under Article 465, or whether it was caused by the abrupt segregation and washing away of the stockpile of the river control,
which makes it a case of avulsion under Article 459, as the case is not between parties as opposing riparian owners contesting ownership over an accession
but rather between a riparian owner and the one in possession of the island.

GOVERNMENT VS. COLEGIO DE SAN JOSE

G.R. No. L-30829 August 28, 1929

THE GOVERNMENT OF THE PHILIPPINE ISLANDS, applicant-appellant,


vs.
COLEGIO DE SAN JOSE, ET AL., claimants.
COLEGIO DE SAN JOSE, appellee.

FACTS:

During the months of September to November every year, the waters of Laguna de Bay cover a long strip of land along the eastern border of the two parcels
of land in question. The claimant Colegio de San Jose contends that the parcels of land are a part of the Hacienda de San Pedro Tunasan belonging it, which
has been in possession thereof since time immemorial by means of its tenants or lessees and farmers. In contrast, the Government contends that the said two
parcels of land belong to the public domain, and its evidence tends to prove that they have always been known as the shores of Laguna de Bay. The CFI
rendered a decision in favor of Colegio de San Jose ordering the registration of the 2 parcels of land in accordance with law. Both admitted that the strip was
formerly covered by water but since the Bay receded, it was now uncovered. The government tried to apply Art. 458 which states the adjoin estate (the
College) does not acquire the land left dry by the natural decrease of the waters.

34
ISSUES:

Whether or not Art. 458 is applicable.

HELD:

No. Article 367 (now Art.458) provides that “the owners of estates bordering on ponds or lagoons, do not acquire the land left dry by the natural decrease of
the waters, nor lose those inundated by them in extraordinary floods.” The provision refers to ponds and lagoons, and has therefore no application to the
present case, which refers to a lake, a lagoon being legally distinct in character from a lake. Instead, Art.77 of the Spanish Law of Waters should apply, which
provides: “Lands accidentally inundated by the waters of lakes, or by creeks, rivers or other streams shall continue to be the property of their respective
owners.” Therefore, they must belong to Colegio de San Jose as part of Hacienda de San Pedro Tunasan, which was originally owned by it.

CELESTIAL VS. CACHOPERO

G.R. No. 142595 October 15, 2003

RACHEL C. CELESTIAL, petitioner,


vs.
JESSE CACHOPERO, respondent

FACTS:

Petitioner Rachel Celestial is the sister of defendant Jesse Cachopero. They had a dispute over a piece of land which was a dried-up creek, as Cachopero
was trying to obtain a Miscellaneous Sales Application (MSA) to the Department of Environment and Natural Resources (DENR) alleging that he had been the
owner of that land whereon he built a house and other improvements. However, Celestial protests that she has preferential right over the land because it is
adjacent to and is the only outlet from her house. According to the Bureau of Land, the land in dispute was a creek and is therefore outside the commerce of
man. The first MSA was denied by the Municipal Trial Court (MTC) prompting Cachopero to obtain another MSA which was granted by the DENR. Due to
conflicting interests of the parties, the land in dispute must be sold in a public auction.

ISSUE:

Whether or not the land in question owned by one of the parties when it is classified as outside the commerce of man

HELD:

No. A dried up creek is property of public dominion and not susceptible to acquisitive prescription

As for Celestial‘s claim of ownership over the subject land, admittedly a dried-up bed of the Salunayan Creek, based on (1) her alleged long term adverse
possession and that of her predecessor-in-interest, Marcelina Basadre, even prior to October 22, 1966, when she purchased the adjoining property from the
latter, and (2) the right of accession under Art. 370 of the Spanish Civil Code of 1889 and/or Article 461 of the Civil Code, the same must fail.

Since property of public dominion is outside the commerce of man and not susceptible to private appropriation and acquisitive prescription, the adverse
possession which may be the basis of a grant of title in the confirmation of an imperfect title refers only to alienable or disposable portions of the public
domain. It is only after the Government has declared the land to be alienable and disposable agricultural land that the year of entry, cultivation and exclusive
and adverse possession can be counted for purposes of an imperfect title.

BAES VS. CA

G.R. No. 108065 July 6, 1993

SPOUSES FELIX BAES AND RAFAELA BAES, petitioners,


vs.
THE COURT OF APPEALS AND REPUBLIC OF THE PHILIPPINES, respondents.

35
FACTS:

In 1962, the Government dug up a canal on a private estate in order to streamline the Tripa de Gallina creek (in other words, there was a mand-made change
of river course). Said private estate was acquired by petitioner Baes, and was subdivided in to three lots. It was lot 2958-C which was totally occupied by the
canal so the Government in exchange granted him a lot near but not contiguous to C. The old river bed was filled up by soil from Lot C. Petitioner now claims
ownership over the old river bed on the basis of Article 461 which says that abandoned river beds belong to the riparian owners whose land is occupied by the
new course of water.

ISSUE:

Whether or not Article 461 applies

RULING:

YES!

If the riparian owner is entitled to compensation for the damage to or loss of his property due to natural causes, there is all the more reason to compensate
him when the change in the course of the river is effected through artificial means. The loss to the petitioners of the land covered by the canal was the result of
a deliberate act on the part of the government when it sought to improve the flow of the Tripa de Gallina creek. It was therefore obligated to compensate the
Baeses for their loss.

We find, however, that the petitioners have already been so compensated. Felix Baes was given Lot 3271-A in exchange for the affected Lot 2958-B through
the Deed of Exchange of Real Property dated June 20, 1970. This was a fair exchange because the two lots were of the same area and value and the
agreement was freely entered into by the parties.

G.R. Nos. L-66075-76 July 5, 1990


EULOGIO AGUSTIN, HEIRS OF BALDOMERO LANGCAY, ARTURO BALISI & JUAN LANGCAY, petitioners,
vs.
INTERMEDIATE APPELLATE COURT, MARIA MELAD, TIMOTEO MELAD, PABLO BINAYUG & GERONIMA UBINA, respondents.

FACTS:
 This case was filed questioning the decision of the trial court which was eventually affirmed by the Court of Appeals.
 As the years went by, the Cagayan River moved gradually eastward, depositing silt on the western bank. The shifting of the river and the siltation
continued until 1968.
 In 1950, all lands west of the river were included in the Solana Cadastre.
 Through the years, the Cagayan River eroded lands of the Tuguegarao Cadastre on its eastern bank among which was defendant-petitioner
Eulogio Agustin's Lot 8457 (Exh. E-Melad), depositing the alluvium as accretion on the land possessed by Pablo Binayug on the western bank.
 However, in 1968, after a big flood, the Cagayan River changed its course, returned to its 1919 bed, and, in the process, cut across the lands of
Maria Melad, Timoteo Melad, and the spouses Pablo Binayug and Geronima Ubina whose lands were transferred on the eastern, or Tuguegarao,
side of the river. To cultivate those lots they had to cross the river.
 In April, 1969, while the private respondents and their tenants were planting corn on their lots located on the eastern side of the Cagayan River, the
petitioners, accompanied by the mayor and some policemen of Tuguegarao, claimed the same lands as their own and drove away the private
respondents from the premises.
 On April 21, 1970, private respondents Maria Melad and Timoteo Melad filed a complaint (Civil Case No. 343-T) to recover Lot No. 3351 with an
area of 5 hectares and its 6.6-hectare accretion. On April 24, 1970, private respondent Pablo Binayug filed a separate complaint (Civil Case No.
344-T) to recover his lots and their accretions.Trial court and Court of appeals rendered their decision in favor of Melad.

ISSUE:

Whether Court of Appeals erred in affirming in the decision of the trial court:

1. in declaring that the land in question had become part of private respondents' estate as a result of accretion;

2. in declaring that the accretion to private respondents' estate which used to pertain to petitioners' estate cannot preclude the private respondents from being
the owners thereof; and

36
3. in declaring that the ownership of private respondents over the accretion is not affected by the sudden and abrupt change in the course of the Cagayan
River when it reverted to its old bed

RULING:
NO.
The finding of the Court of Appeals that there had been accretions to the lots of the private respondents who did not lose the ownership of such accretions
even after they were separated from the principal lots by the sudden change of course of the river, is a finding of fact which is conclusive on this Court. That
finding is supported by Art. 457 of the New Civil Code which provides:
Art. 457. To the owners of lands adjoining the banks of rivers belong the accretion which they gradually receive from the
effects of the current of the waters. (366)
Accretion benefits a riparian owner when the following requisites are present: (1) that the deposit be gradual and imperceptible; (2) that it resulted
from the effects of the current of the water; and (3) that the land where accretion takes place is adjacent to the bank of a river (Republic vs. CA, 132
SCRA 514).
All these requisites of accretion are present in this case.
The private respondents' ownership of the accretion to their lands was not lost upon the sudden and abrupt change of the course of the Cagayan River in
1968 or 1969 when it reverted to its old 1919 bed, and separated or transferred said accretions to the other side (or eastern bank) of the river. Articles 459 and
463 of the New Civil Code apply to this situation.
Art. 459. Whenever the current of a river, creek or torrent segregates from an estate on its bank a known portion of land and
transfers it to another estate, the owner of the land to which the segregated portion belonged retains the ownership of it,
provided that he removes the same within two years.
Art. 463. Whenever the current of a river divides itself into branches, leaving a piece of land or part thereof isolated, the owner
of the land retains his ownership. He also retains it if a portion of land is separated from the estate by the current. (Emphasis
supplied).
In the case at bar, the sudden change of course of the Cagayan River as a result of a strong typhoon in 1968 caused a portion of the
lands of the private respondents to be "separated from the estate by the current." The private respondents have retained the ownership
of the portion that was transferred by avulsion to the other side of the river.

G.R. No. 94283 March 4, 1991


MAXIMO JAGUALING, ANUNCITA JAGUALING and MISAMIS ORIENTAL CONCRETE PRODUCTS, INC.,petitioners,
vs.
COURT OF APPEALS (FIFTEENTH DIVISION), JANITA F. EDUAVE and RUDYGONDO EDUAVE, respondents.
Cabanlas, Resma & Cabanlas Law Offices for petitioners.
Jaime Y Sindiong for private respondents.

FACTS:

 The parties to this case dispute the ownership of a certain parcel of land located in Sta. Cruz, Tagoloan, Misamis Oriental with an area of 16,452
square meters, more or less, forming part of an island in a non-navigable river.
 Private respondents filed with the Regional Trial Court of Misamis Oriental an action to quiet title and/or remove a cloud over the property in
question against petitioners.
 Trial court dismissed the complaint for failure of private respondents as plaintiffs therein to establish by preponderance of evidence their claim of
ownership over the land in litigation. The court found that the island is a delta forming part of the river bed which the government may use to
reroute, redirect or control the course of the Tagoloan River. Accordingly, it held that it was outside the commerce of man and part of the public
domain, citing Article 420 of the Civil Code.
 As such it cannot be registered under the land registration law or be acquired by prescription. The trial court, however, recognized the validity of
petitioners' possession and gave them preferential rights to use and enjoy the property. The trial court added that should the State allow the island
to be the subject of private ownership, the petitioners have rights better than that of private respondent.
 On appeal to the Court of Appeals, respondent court found that the island was formed by the branching off of the Tagoloan River and subsequent
thereto the accumulation of alluvial deposits. Basing its ruling on Articles 463 and 465 of the Civil Code the Court of Appeals reversed the decision
of the trial court, declared private respondents as the lawful and true owners of the land subject of this case and ordered petitioners to vacate the
premises and deliver possession of the land to private respondents.

ISSUE:
Between the one who has actual possession of an island that forms in a non-navigable and non-flotable river and the owner of the land along the margin
nearest the island, who has the better right thereto?

RULING:

37
The court find no error committed by respondent court and DENY the petition for lack of sufficient merit.

It is well-settled that lands formed by accretion belong to the riparian owner. This preferential right is, under Article 465, also granted the owners of
the land located in the margin nearest the formed island for the reason that they are in the best position to cultivate and attend to the exploitation
of the same. In fact, no specific act of possession over the accretion is required. If, however, the riparian owner fails to assert his claim thereof, the
same may yield to the adverse possession of third parties, as indeed even accretion to land titled under the torrens system must itself still be
registered.

Petitioners may therefore, acquire said property by adverse possession for the required plumber of years under the doctrine of acquisitive prescription. Their
possession cannot be considered in good faith, however, because they are presumed to have notice of the status of private respondents as
riparian owners who have the preferential right to the island as recognized and accorded by law; they may claim ignorance of the law, specifically
Article 465 of the Civil Code, but such is not, under Articles 3 and 526 of the same code, an adequate and valid defense to support their claim of
good faith. Hence, not qualifying as possessors in good faith, they may acquire ownership over the island only through uninterrupted adverse
possession for a period of thirty years. By their own admission, petitioners have been in possession of the property for only about fifteen years.
Thus, by this token and under the theory adopted by petitioners, the island cannot be adjudicated in their favor.

This case is not between parties as opposing riparian owners contesting ownership over an accession but rather between a riparian owner and the one in
possession of the island. Hence, there is no need to make a final determination regarding the origins of the island, i.e., whether the island was initially
formed by the branching off or division of the river and covered by Article 463 of the Civil Code, in which case there is strictly no accession
because the original owner retains ownership, or whether it was due to the action of the river under Article 465, or, as claimed by petitioners,
whether it was caused by the abrupt segregation and washing away of the stockpile of the river control, which makes it a case of avulsion under
Article 459.

We are not prepared, unlike the trial court, to concede that the island is a delta which should be outside the commerce of man and that it belongs to the State
as property of the public domain in the absence of any showing that the legal requirements to establish such a status have been satisfied, which duty properly
pertains to the State. However, We are also well aware that this petition is an upshot of the action to quiet title brought by the private respondents against
petitioners. As such it is not technically an action in rem or an action in personam, but characterized as quasi in rem which is an action in
personam concerning real property. Thus, the judgment in proceedings of this nature is conclusive only between the parties and does not bind the State or the
other riparian owners who may have an interest over the island involved herein.

NOTE: A riparian owner refers to a person who owns land bounding upon a river, lake, or other water course.

G.R. No. L-5416 July 26, 1954

ALFREDO MONTELIBANO, ET AL., plaintiffs-appellants,


vs.
THE BACOLOD-MURCIA MILLING CO., defendants-appellants.
San Juan, Africa, Yñiquez and Benedicto and Abundio Z. Arrieta for
plaintiffs-appellants.
Vicente Hilado and Nolan and Manoloto for defendants-appellants.

FACTS:

 Plaintiffs are sugar planters, members of the Bacolod Murcia Planters' Association, Inc., or assignees of sugar planters. The former have contracts
with the defendant corporation, hereinafter known as the Central, for the delivery of their sugar cane to the sugar mill of the defendant for milling
and processing into sugar. In accordance with the contracts, which the planters had signed with the defendant, the sugar processed from the sugar
cane delivered by each planter was to be divided between the planter and the Central in the following proportion, namely, 60% for the planter and
40% for the Central.
 At the time of the occupation of Negros Occidental by the Japanese forces on May 21, 1942, there were on deposit at the Central's warehouse
664,091.22 piculs of sugar, of which 128,452.24 belonged to the plaintiffs, 284,425.81 to the defendant Central, and the balance to planters not
parties to the action
 After liberation (around March to June, 1945) and before the proration above set forth, plaintiff Alfredo Montelibano withdrew from the warehouse
some 12,789 piculs. Of these around 5,115.60 piculs were the share of the defendant Central. Montelibano received a bill of P45,273.06 for the
value of this sugar, and he proposed to pay the said amount in installments. A first payment of P10,000 was made. The amount of the bill was
based on a basic price of P8.85 per picul. The balance of the price has not yet been paid by plaintiff Alfredo Montelibano.
 The present action of plaintiffs is predicated on the claim that the defendant has already been fully paid for its share of the sugar in the warehouse,
as it had sold during the period from April, 1943, to March, 1945, some 284,601 piculs, in excess of around 175.19 piculs of its own share, and had
received the total price of this amount (P2,410,790.03), so that the sugar remaining at the time of the liberation pertained and belonged exclusively
to plaintiffs and the other planters.
 The trial court found that the sugar remaining in the central's warehouse at the time of the liberation was already purchased by the Military
Administration, but it could not withdraw the same by reason of the advent of the liberation; that as the sugar of the parties were all mixed up, none

38
of the owners could claim exclusive ownership of those remaining in the warehouse, and their rights thereto should be governed by the provision of
Article 381 of the Spanish Civil Code.
 Parties plaintiffs and defendant appeal from a judgment of the Court of First Instance of Negros Occidental dismissing plaintiff's complaint for the
recovery of P4,712,501.89, representing the value of sugar alleged to belong to them and existing in defendant's warehouse at the time of the
liberation, and ordering plaintiff Alfredo Montelibano to pay defendant the sum of P35,163.06, plus legal interest thereon from April, 1945, until fully
paid. Plaintiffs appeal from the judgment of dismissal, and defendant from the judgment in so far as it fixes at P35,163.06 as the amount defendant
is entitled to recover from plaintiff Alfredo Montelibano.

ISSUE:

Who are the legal owners of the sugar existing in the Central's warehouse at the time of the liberation?

whether the act of purchase was an act of confiscation of enemy property by the military occupant, or one of requisition, or one of voluntary sale?

RULING:

Affirmed the decision of the trial court.

Irrespective of the legality or illegality of the purchase of plaintiffs' sugar (by the Japanese Military Administration, for which defendant may not
certainly be made responsible, the fact remains that in consequence thereof of warehouse orders for the release of plaintiffs' sugar were issued and sugar
actually taken from the warehouse. Also by the sale of defendants' sugar, release were authorized to the purchaser and withdrawals made. But evidently the
delivery of all the sugar sold by both was not completed, as some 150,000 piculs remained thereafter. As to this sugar (remaining), we hold that title thereto
remained in the original owners, because ownership of personal property sold is not transferred until actual delivery — non nudis pactis, sed traditione dominia
rerum transferuntur. (Fidelity and Deposit Co. vs. Wilson, 8 Phil., 51; Crusado vs. Bustos, 34 Phil., 17.)

It also follows that as the sugar of the plaintiffs and of the other planters and of the Central were stored together in one single mass, without separation of
identification, and as it appears that the Mitsui Bussan Kaisha mad withdrawals of sugar from the Central's warehouse without express statement as to whose
sugar was being withdrawn, whether the planters' or the Central's, it is absolutely impossible, physically or legally, to determine whose sugar it was that
remained after the withdrawals. There is no legal basis for plaintiffs' proposition that as the taking of their sugar was without their consent, and that of the
defendant's with its consent, all that remained is theirs. The only legal solution is, as the mass of sugar in the warehouse was owned in common, and
as it is not possible to determine whose sugar was withdrawn and whose was not, the mass remaining must pertain to the original owners in the
proportion of the original amounts owned by each of them. This is the solution expressly indicated by the law (article 381, Spanish Civil Code), and
the one most consistent with justice and equity.

ART. 381. If, by the will of their owners, two things of identical or dissimilar nature are mixed, or if the mixture occurs accidentally, and in
the latter case the things can not be separated without injury each owner shall acquire a right in the mixture proportionate to the part
belonging to him, according to the value of the things mixed or commingled. (Spanish Civil Code)

ART. 381. If, by the will of their owners, two things of identical or dissimilar nature are mixed or if the mixture occurs accidentally, if in the
latter case the things cannot be separated without injury, each owner shall acquire a right in the mixture proportionate to the part
belonging to him according to the value of the things mixed or commingled.

Lastly, article 393 of the Civil Code, referring to common ownership, provides that the share of the participants in the benefits, as well as in the
charges, shall be proportionate to their respective interests.

This being the rule, it is obvious that whenever an undivided property gains an increase in its area, all the co-owners shall be entitled to participate
in the benefits to be proportionate to their shares; if it suffers diminution they shall have to share, too, the charges in accordance with their interest.
(Tarnate vs. Tarnate, 46 Off. Gaz. (No. 9) 4397,4403-4404)

If goods of the same kind owned by various persons are so mixed with the mutual consent of the owners that the portions or shares of
the various owners in the mixture are indistinguishable, the owners become tenants in common of the mixture, each having an interest
in common in proportion to his respective shares. This is the rule of the civil law. The doctrine finds its most frequent application where
several owners deposit grain in a warehouse although it of course exists wherever the goods of two or more parties are indistinguishably mingled
by common consent, as where quantities of oil belonging to different persons are stored in a tank. In such cases, in the event of partial loss, there
will be prorated distribution of the loss. Where such a confusion arises it seldom causes inconvenience, embarrassment, or dispute, for the
separation of the intermingled goods into the aliquot shares of the owners is merely a matter of measuring, weighing, counting, or selecting, and in
all such cases it is certain that he is entitled to receive back a like quantity. Since they are tenants in common, however, the co-owners are subject
to stand their pro rata share of any loss which may accrue to the general property from diminution, decay, or other causes. (11 Am. Jur. 532-533.)

39
With respect to defendant's counterclaim, we agree with the trial court that the evidence submitted shows that P8.85 is the fair price of the sugar taken by
plaintiff Alfredo Montelibano. Defendant's own original bill fixed this as a price for said sugar (Exhibit 49), and sales made to third persons at the time the sugar
was withdrawn were at prices fluctuating around this sum. We find no reason, therefore, for disturbing the judgment in relation thereto.

G.R. No. L-31163 November 6, 1929


URBANO SANTOS, plaintiff-appellee,
vs.
JOSE C. BERNABE, ET AL., defendants.
PABLO TIONGSON and THE PROVINCIAL SHERIFF OF BULACAN, appellants.
Arcadio Ejercito and Guevara, Francisco and Recto for appellants.
Eusebio Orense And Nicolas Belmonte for appellee.
FACTS:
 This appeal was taken by the defendants Pablo Tiongson and the Provincial Sheriff of Bulacan from the judgment of the Court of First of said
province, wherein said defendant Pablo Tiongson was ordered to pay the plaintiff Urbano Santos the value of 778 cavans and 38 kilos of palay, at
the rate of P3 per cavan, without special pronouncement as to costs.
 On March 20, 1928, there were deposited in Jose C. Bernabe's warehouse by the plaintiff Urbano Santos 778 cavans and 38 kilos of palay and by
Pablo Tiongson 1,026 cavans and 9 kilos of the same grain.
 Pablo Tiongson filed with the Court of First Instance of Bulacan a complaint against Jose C. Bernabe, to recover from the latter the 1,026 cavans
and 9 kilos of palay deposited in the defendant's warehouse. At the same time, the application of Pablo Tiongson for a writ of attachment was
granted, and the attachable property of Jose C. Bernabe, including 924 cavans and 31 1/2 kilos of palay found by the sheriff in his warehouse, were
attached, sold at public auction, and the proceeds thereof delivered to said defendant Pablo Tiongson, who obtained judgment in said case.
 The herein plaintiff, Urbano Santos, intervened in the attachment of the palay, but upon Pablo Tiongson's filing the proper bond, the sheriff
proceeded with the attachment, giving rise to the present complaint.
 It does not appear that the sacks of palay of Urbano Santos and those of Pablo Tiongson, deposited in Jose C. Bernabe's warehouse, bore any
marks or signs, nor were they separated one from the other.
 The plaintiff-appellee Urbano Santos contends that Pablo Tiongson cannot claim the 924 cavans and 31 ½ kilos of palay attached by the defendant
sheriff as part of those deposited by him in Jose C. Bernabe's warehouse, because, in asking for the attachment thereof, he impliedly
acknowledged that the same belonged to Jose C. Bernabe and not to him.

ISSUE:

Whether the trial court erred in ordering the defendant Pablo Tiongson to pay the plaintiff the value of 778 cavans and 38 kilos of palay, the refund of which is
claimed by said plaintiff.

RULING:

The 778 cavans and 38 kilos of palay belonging to the plaintiff Urbano Santos, having been mixed with the 1,026 cavans and 9 kilos of palay belonging to the
defendant Pablo Tiongson in Jose C. Bernabe's warehouse; the sheriff having found only 924 cavans and 31 1/2 kilos of palay in said warehouse at the time
of the attachment thereof; and there being no means of separating form said 924 cavans and 31 1/2 of palay belonging to Urbano Santos and those belonging
to Pablo Tiongson, the following rule prescribed in article 381 of the Civil Code for cases of this nature, is applicable:

Art. 381. If, by the will of their owners, two things of identical or dissimilar nature are mixed, or if the mixture occurs accidentally, if in the
latter case the things cannot be separated without injury, each owner shall acquire a right in the mixture proportionate to the part
belonging to him, according to the value of the things mixed or commingled.

The number of kilos in a cavan not having been determined, we will take the proportion only of the 924 cavans of palay which were attached and
sold, thereby giving Urbano Santos, who deposited 778 cavans, 398.49 thereof, and Pablo Tiongson, who deposited 1,026 cavans, 525.51, or the
value thereof at the rate of P3 per cavan.

Wherefore, the judgment appealed from is hereby modified, and Pablo Tiongson is hereby ordered to pay the plaintiff Urbano Santos the value of 398.49
cavans of palay at the rate of P3 a cavan, without special pronouncement as to costs. So ordered

G.R. No. L-20851 September 3, 1966


JESUS AGUIRRE, petitioner,
vs.
VICTOR S. PHENG, in his capacity as General Manager of the LEONORA & COMPANY, and NATIONAL SHIPYARDS AND STEEL
CORPORATION, respondents.

40
Sisenando Villaluz for petitioner.
M. C. Virata for respondent National Shipyards and Steel Corporation.

FACTS:
 On June 28, 1954, Vicente Aldaba and Teresa V. Aldaba sold to Jesus Aguirre a circular bolted steel tank with a capacity of 5,000 gallons, for the
sum of P900.00, for which the latter delivered to the sellers duly endorsed, Security Bank & Trust Company check No. 281912, in the amount of
P900.00. Aguirre, however, failed to, take physical possession of the tank, having been prevented from doing so by the municipal authorities of Los
Baños, Laguna (where the tank was located), in view of the claim of ownership being made by the Bureau of Public Highways. It appears, however,
that Vicente and Teresa Aldaba again sold the same tank on December 2, 1954 to Zosimo Gabriel, for P900.000. Gabriel, in turn, sold it to the
Leonora & Company on December 5, 1954, for P2,500.00. After some alterations and improvements made on the tank, Leonora & Company was
able to sell the tank to National Shipyards & Steel Corporation (Nassco), for P14,500.00.
 Aguirre immediately filed with Nassco a formal notice of his claim of ownership of the tank, as a consequence of which, payment of the purchase
price to Leonora & Company was suspended. Then, Aguirre instituted two civil cases where the trial court decided that Aguirre was the owner of
the said tank. Defendant National Shipyards and Steel Corporation is hereby ordered to deliver to the said Jesus Aguirre such tank, but in the event
that delivery is not possible, to pay to Aguirre the purchase price of P900.00, and to Leonora and Co. the amount of P11,299.00 which represents
the costs of the improvements made by the said Leonora & Co. In the event that the National Shipyards and Steel Corporation shall deliver the oil
tank to Jesus Aguirre as it is, the latter shall pay to Leonora and Co. the amount of P11,299.00 which, as already stated, was spent by Leonora and
Co. for the improvement of the tank.
 The present case.—On January 9, 1963, the Court of Appeals rendered decision affirming the judgment of the lower court in Civil Case No. 27988,
to return to intervenor Aguirre the sum of P900.00 in case delivery of the tank to him will not be possible —because this was all the amount that
Aguirre had parted with when he purchased said tank. It was Leonora & Co. who had 5 spent the sum of P11,299.00 for the rehabilitation of said
tank and against this amount Aguirre has no rightful claim whatsoever. Of course, in the event of delivery of the tank to Aguirre as improved, it
would be just for him to reimburse Leonora & Co. the sum of P11,299.00. The trial court, therefore, acted properly in denying Aguirre's claim to be
paid the fair and reasonable value of the tank as improved in case the same could no longer be delivered to him.

ISSUE:

Whether the judgment of the Court of Appeals, ordering the return to him of the sum of P900.00 (when the value of the property is at least
P14,500.00), is correct?

RULING:

It is clear that we have here a case of accession by specification: Leonora and Company, as purchaser acting in good faith, spending P11,299.00 for
the reconditioning of the tank which is later adjudged to belong to petitioner Aguirre. There is no showing that without the works made by Leonora &
Company, the tank in its original condition when Aguirre paid P900.00 therefor, would command the price of P14,500 which Nassco was willing to pay.
Although ordinarily, therefore, Aguirre, as owner of the tank, would be entitled to any accession thereto, the rule is different where the works or
improvements or the accession was made on the property by one who acted in good faith. And, it is not contended that the making of the
improvements and incurring of expenses amounting to P11,299.00 by Leonora & Company was done in bad faith. Furthermore, to uphold petitioner's
contention that he is entitled to the sum of P14,500.00 the price of the tank in its present condition, would be to allow him to enrich himself at the expense of
another. The lower courts, therefore, acted correctly in ordering the reimbursement to Leonora & Company of the expenses it made on the
tank.1awphîl.nèt

It must also be remembered that the judgment in Civil Case No. 24914 of the Court of First Instance of Manila, wherein Nassco was directed to pay to Aguirre
the amount of P900.00, in case delivery of the same tank is no longer possible, has already become final. This ruling cannot be disregarded in the present
proceeding which involves the same parties and practically the same issue, arising from the same set of facts.

Nassco cannot also be compelled to pay more than P14,500.00 for the tank, the bid offered by Leonora & Company and accepted by this buyer, and which
must be the actual market value of the property at the time of its delivery to the latter. It has nothing to do at all with the various transactions or sales and the
deprivation of Aguirre's right to possession of the tank, which culminated in this legal suit.

Wherefore finding no error in the decision of the Court of Appeals under review, the present petition is hereby dismissed, with costs against the petitioner. So
ordered.

41

You might also like